All Exams  >   GMAT  >   35 Days Preparation for GMAT  >   All Questions

All questions of Introduction for GMAT Exam

Clearly, General Motors is spending significantly less on marketing its convertible car Volt than Toyota is spending on marketing its car Toyota Prius. Even though both cars were launched roughly at the same time, offer the same functionality and come at the same price range, Prius has sold more than four times as many cars until now as Volt has. This is despite the fact that Volt recently won the prestigious “Car of the Year” award.The above argument is most vulnerable to which of the following criticisms?
  • a)
    It does not take into consideration the perceived value of the “Car of the Year” award.
  • b)
    It assumes without any basis that all cars with the same functionality and price range should sell equally.
  • c)
    It does not consider that it could be a conscious decision of the management at General Motors to spend less money on the marketing of Volt in the beginning and instead use the money to expand dealerships.
  • d)
    It fails to consider that there could be other possible causes for the difference in the sales of Volt and Prius cars.
  • e)
    It fails to take into account other cars launched by Toyota during the same time.
Correct answer is option 'D'. Can you explain this answer?

EduRev GMAT answered
Argument Analysis
Pre-Thinking
Conclusion Clarification
This is a causal argument.
The conclusion states that General Motors is spending significantly less on marketing its convertible car Volt than Toyota is spending on marketing its car Toyota Prius. The conclusion is based on the fact that despite several similarities such as time of launch, functionality and price and the fact that Volt has won the prestigious ‘car of the year award’, the Prius has sold 4 times as many cars as Volt has. Clearly, the author thinks the difference in the sales of cars is due to spending on marketing.
Pre-Thinking Approach
To pre-think a flaw in the reasoning, let’s see how we can falsify this conclusion.  To do so, we will look at the logical structure, focusing on linkage 1 and the conclusion.  We will understand the condition under which the conclusion is falsified. Thus, the flaw would a logical gap in author’s reasoning while reaching the conclusion.
Linkage#1
  • Falsification Condition:  What if General Motors expenditure on the marketing of Volt is the same as Toyota’s spending on the marketing of Prius and some other factor has led to the difference in the number of card sold? For instance, it’s possible that the marketing techniques opted by Toyota are more impactful than the techniques opted by GM. In this case, even though the expenditure is the same, Prius can sell more cars due to the effective marketing plan and thus the conclusion will not hold.
  • Flaw:  The argument is vulnerable to the criticism that it fails to consider any other causes that might have led Prius to sell more than four times as many cars till now as Volt has.
With this pre-thinking in mind, let’s evaluate the answer choices.
Answer Choices
A
It does not take into consideration the perceived value of the “Car of the Year” award.
Incorrect - No flaw
The author mentions this award and says that Prius sold more units in spite of Volt’s receiving this award. Now if we say that the perceived value of the car was low, we cannot argue that this factor could have led to the difference in the sales as the argument doesn’t provide any link between a car’s winning an award and its sales.
B
It assumes without any basis that all cars with the same functionality and price range should sell equally.
Incorrect - No flaw
The author is only attributing a cause for the vast difference in sales. By no means, this attribution indicates that the author assumes that the cars should have sold equally. Also, the scope of the discussion is limited to just these two cars and not “all” cars.
C
It does not consider that it could be a conscious decision of the management at General Motors to spend less money on the marketing of Volt in the beginning and instead use the money to expand dealerships.
Incorrect - Irrelevant
First of all, we don’t even know whether there was a difference in the expenditure. Secondly, the argument is not about what led to the less spending but instead what led to the difference in sales.
D
It fails to consider that there could be other possible causes for the difference in the sales of Volt and Prius cars.
Correct
This choice is written along the lines of our pre-thinking per Linkage #1.  It states that there could be some other factors such as company’s strategy or marketing plan that might have led to the difference in sales of Volt and Prius cars. The argument ignores any such possibility.
E
It fails to take into account other cars launched by Toyota during the same time.
Incorrect - Irrelevant
This option tells us that the argument fails to consider other cars launched by Toyota during the same time. As the argument is about Volt and Prius, not taking other cars into account is not a basis for criticism

Linda: In the 1800s, it was found that one in every six women who gave birth in hospitals died of a fever they had contracted after delivering the child and that the mortality was not as high if they gave birth at home with the help of a mid wife. It was found that the doctors had a poorer sense of hygiene and that their dirty hands and instruments were leading to pathogens entering a woman's bloodstream. Thankfully, hygienic conditions today are much better and women are safer. 
Fiona: But doctors today are so overworked that a number of doctors, while aware of the need for better hygiene, barely find the time to wash their hands. The likelihood of infections caused by doctors is probably not any better. 
Q. Which of the following can be used by Fiona to further establish that Linda need not be correct in her reasoning?
  • a)
    It is reported that around 50,000 people die every year in our country from hospital acquired infections. 
  • b)
    Compulsory hand cleansing is strictly adhered to by all doctors and nurses before every major surgical procedure. 
  • c)
    During a study, doctors handling newborn babies self reported a hand cleansing rate of 90℅.
  • d)
    Nurses are not any better than doctors in adhering to strict cleansing schedules, especially when busy.
  • e)
    A nationwide survey found that doctors, especially experienced ones, are more prone to the belief that they could not possibly be carrying pathogens.
Correct answer is option 'E'. Can you explain this answer?

Meera Rana answered
Step 1 of solving this GMAT CR Question: 
  • Analyzing the Argument Look for differences across the answer options to identify what is being tested in the sentence. Linda’s argument has a cause-effect relationship and a comparison element as well. 
    (i) The causal argument is that the safety of women (or low mortality among women) is dependent on the hygiene awareness among the doctors. 
    (ii) The comparison element is that women today are better off than the women in the 1800s. 
  • If Fiona is to question Linda’s argument, she has to establish that one of these two (the causal and the comparison) relationships do not actually hold true. This is the reason that Fiona’s argument tries to establish that awareness is not enough. Mortality is also affected by the actual practices of the doctors. 
  • To further weaken Linda’s argument, the correct answer Option must extend this discussion or question whether women today are actually better off than the women from the 1800s.
Step 2 of solving this GMAT CR Question: Eliminating Options 
  • Option (A) can be eliminated because we do not know whether these hospital-acquired infections were because of poor practices of the doctors or because of other factors. 
  • Option (B) can be eliminated because the option adds further support to Linda’s argument, when in fact the objective is to weaken her argument. 
  • Option (C) can be eliminated because the option again adds support to Linda’s argument. Some people wonder about the usage of the words “self-reported”, which might imply that the 90% figure may not be accurate. However, it cannot be assumed that the doctors are lying or that the actual rate is much lower than 90%. 
  • Option (D) seems to imply that the nurses do not have proper practices in place. At first glance, the option seems to work. However, the option assumes that doctors do not adhere to the schedules, implying that nurses are as bad. Nurses and doctors could both be GOOD at sticking to the cleansing schedules. Moreover, the focus of the discussion is on the practice of doctors and not of nurses. 
  • Option (E) works because it establishes another reason for carelessness among doctors when it comes to hygiene. If doctors believe they do not carry pathogens, they are more likely to be careless about cleansing schedules.

A recently published paper concludes that tenured professors or those on their way to tenure don’t enhance student learning as much as full-time lecturers outside the tenure system. Clearly, the difference between professors in the tenure system and other full-time lecturers has to do with the reward system for the former. The criterion for rewarding tenured faculty typically places a greater emphasis on research than teaching.
The above argument is based on doubts on the validity of which of the following statements?
  • a)
    A tenured professor is expected to spend greater time on research activities than on teaching.
  • b)
    Non-tenured full time lecturers are rewarded more for the research that they do than for their teaching assignments.
  • c)
    The universities are bound by the law to continue with their existing reward systems.
  • d)
    Generally, a tenured professor is much more experienced than a non-tenured full time lecturer.
  • e)
    No non-tenured professor focuses more on the research activities than teaching in order to become tenured.
Correct answer is option 'B'. Can you explain this answer?

Gauri Iyer answered
In order to doubt the validity of the argument we have to select the statement which can directly counter argue the argument.
Of options, B states that non-tenure full time lecturers are rewarded more than research which is usually going against the argument stated in the passage above where the argument states tenured faculty typically places a greater emphasis on research.
Option A,C,D are OFS
Option E is an strengthener

Analyst 1: Many parents take their children along when they go for shopping for toys because they want to know whether the children will really enjoy playing with a toy before buying it for them. But in many cases, children ask for toys that are either out of their parents’ budgets or are somewhat unsafe for the children. Either way, it is not a good idea to take children along when shopping for toys.
Analyst 2: But since parents have the ultimate authority to decide whether to purchase a particular toy, your consideration is not valid.
Which of the following, if true, most helps to strengthen the conclusion drawn by analyst 1?
  • a)
    Children of a specific age are more adamant about buying a toy when compared to children in other age groups.
  • b)
    Parents are likely to give in to children’s demands, even when the implications of these demands are not favorable.
  • c)
    Children also play with toys that they are not adamant about wanting to buy.
  • d)
    The reason children prefer toys that are either unsafe for them or are out of their parents’ budgets is that most such toys are advertised heavily.
  • e)
    Parents hardly take children along for shopping trips to stores that sell products other than toys.
Correct answer is option 'E'. Can you explain this answer?

Here, Analyst1 concludes that it is not a good idea to take children along when shopping for toys because of the demand of children that can be harmful for either the parent or the children.

Now if parents tend to agree to their childerns' demand and buy them the toys, then the argument of Analyst1 that it is not agood idea to take children for toy shopping holds good. As parents actually end up meeting childrens' demand of harmful toys rather than taking the ultimate authority in rationally deciding whether to buy the toys.

Therefore, option B makes more sense.

Which of the following affects the Dividend Decision of a company?
  • a)
    Taxation Policy
  • b)
    Cash Flow Position
  • c)
    Earnings
  • d)
    All of these
Correct answer is option 'D'. Can you explain this answer?

Aditya Mishra answered
This is so because a company' realise the every ascept of company it totally depends on the other factors of income like earning cash inflow outflow and taxation policy

Analyst: The Mayor’s petition to revise existing pension systems has raised a significant amount of protest from the city’s labor unions, whose leaders fear that the revision will lead to reduced pensions for retired workers, citing as evidence the fact that the pay scales of government workers are often reduced when the economy experiences a downturn. However, in this case, the union leaders’ fears are baseless since the pension amounts have already been guaranteed to government workers.
The analyst’s argument depends on which of the following assumptions?
  • a)
    Some benefits for government employees will continue regardless of the actual amount of pension that is paid to them.
  • b)
    The union leaders’ protests are a result of expectations from pensioners that their interests will be protected by the labor unions.
  • c)
    A current economic downturn is the reason for the Mayor’s decision to revise the city’s existing pension system.
  • d)
    The pay scales of government workers that are reduced in times of economic downturns are not already guaranteed by the government.
  • e)
    Revisions of pension systems have not led to disadvantages for pensioners in the past.
Correct answer is option 'D'. Can you explain this answer?

Amrutha Jain answered
Pre-Thinking
Conclusion Clarification
The conclusion says that the fears of LU leaders are baseless. This means that the author concludes that the revision of PS won’t lead to reduced pensions for RW.
Pre-Thinking Approach
To pre-think assumptions, we will falsify the conclusion or look for scenarios under which it is no longer valid even when the given premises are true. The assumption then will be that such conditions don’t exist.
Linkage #1: Pension amounts already guaranteed to GW
What if, already guaranteed amounts can still change? Let’s say the government which promised these amounts has changed. The new government may not have the obligation to keep the promises made earlier.
Assumption 1: Guaranteed pension amounts can’t change.
Also, the conclusion here discards the comparison with the pay scales of GW because the amounts in case of RW are guaranteed. So, the author is assuming that:
Assumption 2: Pay scales of GW that are reduced under recession aren’t guaranteed.
With this understanding in mind, let’s take a look at the option statements.
Answer Choices
A
Some benefits for government employees will continue regardless of the actual amount of pension that is paid to them.
Incorrect - Irrelevant
The argument is only concerned with pension amounts, not other benefits.
B
The union leaders’ protests are a result of expectations from pensioners that their interests will be protected by the labor unions.
Incorrect - Irrelevant
Reasons behind the union leaders’ protests are irrelevant.
C
A current economic downturn is the reason for the Mayor’s decision to revise the city’s existing pension system.
Incorrect - Irrelevant
The reference to the revisions of the pay scales of GW in times of recession is only an example. It is not a must that the reason behind this revision of PS is the same.
D
The pay scales of government workers that are reduced in times of economic downturns are not already guaranteed by the government.
Correct
This choice is written along the lines of our pre-thinking per Linkage #1, Assumption 2.
E
Revisions of pension systems have not led to disadvantages for pensioners in the past.
Incorrect - Irrelevant
We aren’t concerned with the past. The argument is about the mayor’s plan to revise the PS at present.
 

Recently, a plane crashed near the outskirts of Berlin within minutes after its takeoff from Berlin airport. Initial analysis confirmed that the crash was caused by either the usage of substandard fuel or a fault in the control board. Since the airport staff tested the control board before the plane took off, analysts hypothesize that the reason behind the plane crash was substandard fuel.
Which of the following options, if true, most strengthens the analysts’ hypothesis?
  • a)
    The black box, upon inspection, was found to belong to a generation of models more compatible with fighter planes than with passenger planes.
  • b)
    Upon enquiry, the staff at Berlin airport confirmed that they indeed checked the control board before the plane took off.
  • c)
    Using a low quality fuel can lead to engine failure and a subsequent plane crash.
  • d)
    All the testing conducted by the airport staff is done very extensively to avoid any lapses.
  • e)
    The plane belongs to the ZX3V series which contains a fuel combustion engine that is notoriously famous for explosions within minutes after takeoff.
Correct answer is option 'D'. Can you explain this answer?

Mehul Nair answered
GMAT Critical Reasoning: Strengthen the Argument

The question asks to strengthen the hypothesis that substandard fuel caused the plane crash. The correct answer will provide evidence that supports the claim that substandard fuel was responsible for the crash.

Strengthening Option: D

Option D strengthens the hypothesis that substandard fuel caused the plane crash by providing evidence that the airport staff conducts extensive testing to avoid any lapses. This implies that if there was a problem with the control board, it would have been detected during testing. Therefore, the only other possible explanation for the crash would be substandard fuel, which is consistent with the initial analysis.

Other Options:

Option A: This option is irrelevant to the hypothesis as it talks about the black box belonging to a generation of models more compatible with fighter planes than with passenger planes.

Option B: This option confirms that the staff at Berlin airport checked the control board before the plane took off, but it does not provide any evidence regarding the fuel quality.

Option C: This option provides information on how low-quality fuel can lead to engine failure and a subsequent plane crash, which supports the hypothesis. However, it does not provide any new evidence to strengthen the hypothesis.

Option E: This option talks about the plane belonging to a series that contains a fuel combustion engine that is notoriously famous for explosions within minutes after takeoff. However, it does not provide any direct evidence to support the hypothesis that substandard fuel caused the crash.

In conclusion, option D is the correct answer as it provides evidence that supports the hypothesis that substandard fuel caused the plane crash.

Customers are aware that most of the faults that arise in their mobile phones are due to the excessive exposure of these phones to moisture or water. Accordingly, engineers at Talkit Inc. have developed a water-resistant phone that can stay underwater for at least 30 minutes without any resultant damage to either its hardware or software. However, since the manufacturing cost of the phone is anywhere between 25-30% more than that of an ordinary phone, trade analysts have predicted that the phone will be very expensive and as such not be able to find enough customers. 
Which of the following, if true, argues in favor of the assertion that the analysts’ prediction is premature?
  • a)
    The manufacturing cost of the water-resistant phone is less than that of most high-end smartphones.
  • b)
    The average repair cost of a regular phone in its lifetime is anywhere from 30%-35% of its original selling price.
  • c)
    According to several recent surveys of consumers, people looking to purchase a new phone are unlikely to buy models that are more expensive than most popular models.
  • d)
    The latest tests done at Talkit Inc. have indicated that the water-resistant phone can actually withstand at least forty minutes of remaining underwater.
  • e)
    The phone has a built-in high-resolution camera that can match if not beat the quality of the usual 8-megapixel camera the regular phones come with.
Correct answer is option 'E'. Can you explain this answer?

Ashwitha K S answered
Why isn't Option B an Apt answer ??

While surgical tools are constantly being upgraded with the objective of making surgeries less invasive, a recent study has shown that in one out of every 300 cases, surgery performed using the power morcellator, a newly-developed tool used in treatment of uterine cancer, has actually resulted in the spread of the cancer in the patient’s body. However, the Center for Disease Control (CDC) plans to continue to allow the use of morcellators in oncological treatment as long as physicians stringently select the patients to whom such treatment is administered.
Which of the following would be most useful in assessing the CDC’s plan?
  • a)
    Whether banning the use of a surgical tool is a drawn-out process as it requires multiple levels of authorization
  • b)
    Whether patients are aware of the risks associated with surgical procedures performed using minimally invasive techniques
  • c)
     Whether some patients will continue to benefit from surgical treatments performed using power morcellators.
  • d)
    Whether the effects of a surgical procedure can be determined before the procedure is performed.
  • e)
    Whether other surgical tools currently in use have been known to cause negative side effects, but have not been banned by the CDC. 
Correct answer is option 'D'. Can you explain this answer?

Assessing the CDC's Plan

Effects of Surgical Procedure
- One of the most useful factors to consider in assessing the CDC's plan is whether the effects of a surgical procedure can be determined before it is performed. This is crucial in the case of power morcellators, as the risk of spreading cancer cells can have serious consequences for the patient.
- Knowing whether there are reliable methods to predict the outcomes of surgeries using morcellators can help determine the feasibility and safety of continuing their use in oncological treatment.
By evaluating the ability to assess the effects of surgical procedures beforehand, the CDC can make informed decisions about the use of power morcellators in cancer treatment.

A person is considered obese if her BMI i.e. Body Mass Index, a ratio of a person’s weight to square of her height, exceeds a certain number. In 1990, the average BMI of the population was just below the threshold level for obesity. However, in a recent survey conducted, it was found that the average weight was 10% more than that of the population in 1990. Given that there has not been any increase in average height of the population since 1990, the obesity levels in the country must have increased.
Which of the following, if true, would cast the strongest doubt on the conclusion drawn above?
  • a)
    In another survey conducted, it was found that the average fat consumption by individuals in the country has decreased.
  • b)
    A major criterion for selecting participants in the survey was the willingness of the participants to actively participate in the survey.
  • c)
    The population of country has increased by more than 10% since 1990.
  • d)
    Obese people have a shorter life span than other people.
  • e)
    All the participants in the survey were taller than an average citizen of the country   
Correct answer is option 'E'. Can you explain this answer?

It's a comparison type and generalisation kind CR Argument.
so the natural assumptions just after reading the passage are

what happened in the past is similar to present day survey.that means the measurement criteria has not changed.

survey word is used ,so another assumption will be

The survey sample is representative in both conditions.

so let's go to the choices
A says about fat ,what's the outcome do you know.we are sure it .ans is no I am not sure.so out

B says all the participants are having heigher height than the rest and the conclusion says the BMI MUST HAVE INCREASED IN PASSAGE.AND THIS COUNTERS THE CLAIM,bkz weight increased by 10% and heights of the sample space also increased above the average, but not given how much.and this is point that make the conclusion doubtful bkz the conclusion uses the word MUST increase.a slightest doubt creating but non extensive answer choice can be good strengthener or weakner.

c says nonsense.ha ha
d says obese have short life span so what do with bmi

e says cunning thing :it says major criteria for selection is willingness,now the question is who will have the willingness,and for godsake don't extrapolate to think that only higher weight to lower height people will agree to participate bla bla bla..
Answer is B,based the assumption that the survey sample is representative or its called the specific to generic assumption CR type.

In an attempt to improve the economic and social welfare of the people, the government has recently announced a scheme under which it will provide certain staple foods at 30% discount to the authorized dealers. The reduction in the prices of these foods will significantly improve the economic conditions of the people from low income groups.
Which of the following would be most useful to establish in evaluating the argument?
  • a)
    Whether discounting the prices of staple foods is the only way to improve the economic and social welfare of the people
  • b)
    Whether people with medium or high income groups will also benefit from the scheme
  • c)
    Whether the expenditure on staple foods under question forms a significant chunk of the income of people from low income groups
  • d)
    Whether the people from low-income groups will appreciate some other kind of financial assistance more than the one being offered by the government
  • e)
    Whether this scheme would have an adverse impact on the budgetary allocation of other welfare initiatives taken by the government.
Correct answer is option 'E'. Can you explain this answer?

Meera Rana answered
 Option ( E) is the correct answer. 
Explanation:-
Option A) This is too extreme. We are not considered whole people. We are concerned only about low income people. 
Option B) Who cares about medium or high income groups. We are concerned only about low income people. 
Option C) If they say yes / no this doesn't give details  where low income people are benefitted . 
Option D)This is irrelevant. 
Option E)  If yes, only then the low Income people will be benefited. 
 
So,  option ( E) is the  right one.

A recent examination of the waters of the Southern Ocean showed a substantial increase in pollution levels, which is usually caused by routine oil spills during industrial drilling operations in the area. Last month, an article published in the biannual newsletter of a prominent environmental organization attributed this increase to the operations of Gaston Inc., an oil company with several large deep-water drilling operations. This claim is not undermined by the finding that none of Gaston Inc.’s oil plants in the Southern Ocean had any oil spills, because ____________.
  • a)
    most increases in the pollution levels of oceans take place because of deep-water drilling operations
  • b)
    oil usually floats several feet below the surface of the ocean rather than at the surface itself
  • c)
    pollutants that enter the ocean sometimes take decades to affect the marine environment
  • d)
    tankers that transport oil from drilling plants to offshore locations often develop leaks
  • e)
    calculated amounts of oil are sometimes deliberately discarded by oil plants to maintain water pressure in the area surrounding the plant
Correct answer is option 'D'. Can you explain this answer?

Mihir Nambiar answered
Explanation:

The argument presented states that the increased pollution levels in the Southern Ocean are caused by routine oil spills during industrial drilling operations, and specifically blames Gaston Inc. for this. The fact that none of Gaston Inc.'s oil plants had any oil spills is presented as a potential counterargument, but the correct answer explains why this does not undermine the original claim.

The correct answer is D, tankers that transport oil from drilling plants to offshore locations often develop leaks. This is a valid reason why pollution levels could increase even if there were no oil spills directly from the drilling plants themselves. The following points explain this further:

Tankers transport oil from drilling plants to offshore locations:

- Tankers are used to transport oil from drilling plants to offshore locations where it is further processed or shipped to other locations.

Tankers often develop leaks:

- Tankers are known to develop leaks due to various reasons such as corrosion, accidents, or deliberate dumping of oil to reduce the weight of the tanker and increase its speed.

Leaked oil can contribute to pollution levels:

- When oil leaks from tankers, it can contribute to the pollution levels in the ocean. This is because the leaked oil can spread over a large area and affect marine life and the environment.

Therefore, the fact that none of Gaston Inc.'s oil plants had any oil spills does not undermine the claim that their operations contributed to the increased pollution levels in the Southern Ocean, as leaks from tankers could also be responsible for this.

A restaurant that comes up with an innovative dish can normally increase its profits by capitalizing on the novelty of the dish. Since recipes can be easily replicated, the restaurant with a new dish advertises it aggressively to attract customers while the novelty factor can still be used as a selling point. But aggressive advertising generates a fear of losing customers in the competitors and gives them a strong incentive to quickly replicate the new dish. Therefore, the strategy to maximize overall profit from a new dish is to advertise it selectively.
Which of the following, if true, strengthens the suggestion given by the author?
  • a)
    The patrons of a restaurant are the ones who are most likely to try a new dish at the restaurant. 
  • b)
    The word of mouth publicity generated by the customers of the restaurant is a good enough reason for the competitors to replicate the dish.
  • c)
    The recipes for the innovative dishes are usually kept under close guard by the restaurants that develop the dishes.
  • d)
    Aggressive marketing usually generates far more profits than does selective marketing.
  • e)
    Selective marketing has been proven to be an effective strategy for fast food restaurant chains.
Correct answer is option 'E'. Can you explain this answer?

Jatin Kapoor answered
Explanation:

Selective marketing has been proven to be an effective strategy for fast food restaurant chains
- This option directly supports the author's suggestion that selectively advertising a new dish is the best strategy to maximize overall profit.
- If selective marketing has already been proven to be effective for fast food restaurant chains, it lends credibility to the author's argument that this strategy can also work for other types of restaurants.
- This strengthens the suggestion by providing evidence that selective marketing is a successful approach in the restaurant industry.
Therefore, option E is the correct answer as it reinforces the idea that selectively advertising a new dish can lead to increased profits for a restaurant.

Consumers appreciate free products more than they appreciate discounted products when such products are paired with full priced high-end products. The reason behind the same could be the monetary value they attach to various goods. Research shows that consumers tend to believe that the monetary value of the free product is probably consistent with the value of the high end purchased product with which it is being offered for free. However, when a discounted product is paired with a high-end product, consumers attach less monetary value to it than they would have had the same product been offered for free.
Which of the following best supports the conclusion as stated above?
  • a)
    Before making a purchase decision, consumers thoroughly check the price of the products with which free or discounted products are paired.
  • b)
    It is highly unlikely that consumers would ever actually buy most of the products offered for free with the high-end purchased products.
  • c)
    Consumers are aware that the price of the product offered for free or on discount never exactly matches the cost of the high-end product to the retailer.
  • d)
    In an experiment, the customers of a gold jewelry shop who were given a free bottle of wine with a certain purchase expressed more satisfaction than the customers who were offered the same wine for one dollar when they made the same purchase.
  • e)
    Customers in general are aware of the actual price of the products even when these products are offered for free or on discount.
Correct answer is option 'D'. Can you explain this answer?

Anirban Das answered
Understanding the Conclusion
The conclusion states that consumers value free products more highly than discounted products, especially when they are associated with high-end items. This is attributed to the perceived monetary value consumers attach to these products.
Support from Option D
Option D provides a clear example that substantiates the conclusion:
- Experiment Context: Customers at a jewelry shop received either a free bottle of wine or were offered the same wine for one dollar with their purchase of gold jewelry.
- Consumer Satisfaction: The customers who received the wine for free reported greater satisfaction compared to those who paid for it, even at a low price.
Key Insights
- Perceived Value: The free wine is perceived as a bonus, enhancing the overall value of the high-end jewelry purchase. This aligns with the conclusion that consumers believe the free product holds value consistent with the high-end item.
- Comparison with Discounts: In contrast, offering a discounted product (even at a low price) does not elevate its perceived value in the same way. Consumers might associate the discount with lower quality, diminishing their overall satisfaction.
Conclusion Reinforcement
The experiment supports the idea that consumers derive more satisfaction from free offerings than from discounted ones, especially in a luxury context. This reinforces the conclusion that the perceived monetary value of the products influences consumer preferences and satisfaction levels. Thus, the findings in Option D effectively validate the conclusion drawn in the original statement.

Although dried roots in well-watered orchids are caused primarily by the deficiency of a chemical compound called Vitadex, researchers believe that the presence of Ditalex in the soil could also be a cause of the same. Traces of Ditalex were found in 70 percent of the orchids with dried roots, but in only 10 percent of the orchids without dried roots. Ditalex was accidentally added to a few pesticides last year and researchers hypothesize that these affected orchids are from farms that once used this particular batch of pesticides.
Which of the following, if true, most strongly supports the researchers’ hypothesis?
  • a)
    Regulations in the pesticides manufacturing industry have now banned the usage of Ditalex.
  • b)
    Recently tested samples of the pesticides used last year have revealed traces of Ditalex.
  • c)
    On farms where the infected pesticide was never used, the samples of orchids that had dried roots did not contain Ditalex.
  • d)
    In some cases, orchids exposed to Ditalex are not affected by dried roots.
  • e)
    There is no direct proof that dried roots are primarily caused by Vitadex.
Correct answer is option 'C'. Can you explain this answer?

Argument Analysis
Pre-Thinking
Conclusion Clarification
The argument contains 2 independent conclusions referred to as “Researchers’ Belief” and “Researchers’ hypothesis”.  However, if you refer to the question stem, it says “Which of the following, if true, most strongly supports the researchers’ hypothesis?”. Therefore, the question is referring to the “Researchers’ hypothesis” only.
Pre-Thinking Approach
Let’s see how we can increase the belief in the conclusion.  To do so, we will look at the logical structure, focusing on linkage 1 and the conclusion (researchers’ hypothesis). 
When we look at the logical structure, we see that the hypothesis talks about a possible cause of Ditalex found in some orchids.  Here the possible cause refers to the usage of Ditalex containing pesticides in the farms that grew the orchids.
Linkage#1
Strengthener – A reasoning that tells that: Some other factor did not lead to Ditalex in these orchids. (alternate cause)
  • For example, preliminary test reveal that ABC chemical, which reacts with certain chemicals in the soil to form Ditalex, was not present in the areas where the orchids containing Ditalex were found.
Strengthener – Evidence that shows: Ditalex was not found in farms that did not use the batch of pesticides that contained Ditalex. (no cause, then no effect)
  • For example, no complaint was received regarding Ditalex in orchids that were grown in other farms.
With this understanding, let us evaluate the option statements.
Answer Choices
A
Regulations in the pesticides manufacturing industry have now banned the usage of Ditalex.
Incorrect - Irrelevant
We already know that Ditalex was accidentally used in pesticides in a particular batch. All we are trying to validate is – whether the same batch of pesticides led to Ditalex in certain orchids that were grown in the farms that used the pesticide.
B
Recently tested samples of the pesticides used last year have revealed traces of Ditalex.
Incorrect - no new information
This option does not introduce any new information since we already know that the batch of pesticides contained traces of Ditalex.
C
On farms where the infected pesticide was never used, the samples of orchids that had dried roots did not contain Ditalex.
Correct
It matches the second strengthener that we identified in the pre-thinking analysis
D
In some cases, orchids exposed to Ditalex are not affected by dried roots.
Incorrect - does not support the hypothesis
This option does not address the causal relationship that the hypothesis is trying to establish. 
E
There is no direct proof that dried roots are primarily caused by Vitadex.
Incorrect - Irrelevant
This option does not talk about the relationship that pesticides containing Ditalex used in farms led to Ditalex in orchids that grew there.

Several studies conducted over a decade have shown that most of the people in the mountain states of the US listen to music primarily in an active manner: i.e. they listen specifically to listen rather than being in a space where music is playing in the background. On the other hand, the majority of people in the coastal states of the US primarily consume their music in a passive manner. As they are trying to attract as many active listeners as possible, record labels should conduct maximum marketing activities in the mountain states and not in the coastal ones.Which of the following indicates a flaw in the author’s reasoning?
  • a)
    The author doesn’t take into account the possibility that there might be far fewer restaurants and pubs, places where music plays in the background, in the mountain states.
  • b)
    The author doesn’t consider listeners who consume their music in both possible ways.
  • c)
    Coastal states can have more active listeners than mountain states.
  • d)
    The author does not account for the fact that listening to music in an active way in the mountain states may only be a passing phase.
  • e)
    Word of mouth recommendations can sometimes be more effective than marketing activities.
Correct answer is option 'C'. Can you explain this answer?

Aditya Sharma answered
Flaw in the Author's Reasoning:

Incorrect Assumption:
- The author assumes that the majority of people in the coastal states consume music passively, without considering the possibility that coastal states could have more active listeners than mountain states.

Counterpoint:
- It is possible that there are thriving music scenes in coastal states, with a high number of music enthusiasts who actively engage with music. These states may have vibrant music communities, venues, and events that attract active listeners.

Limitation of the Study:
- The studies mentioned in the argument may not have considered all factors that influence music consumption habits. Factors such as cultural differences, demographic trends, and regional music preferences could impact how people in different states engage with music.

Conclusion:
- The author's conclusion that record labels should focus marketing activities in the mountain states based on the assumption of passive music consumption in coastal states is flawed. It is essential to consider the diversity of music consumption habits across different regions and not make broad generalizations based on limited studies.

The red tramfish is an extremely rare species of fish that can be found only in some of the lake shores of the Silver Lakes region. A recent study has found that the oil extracted from the bones of the red tramfish can be used to develop an extremely effective medicine for arthritis. Since this finding has led to a sudden interest in this fish, the locals fear that the species will soon be extinct. 
Which of the following, if true, supports the idea that the fear of the locals may not be misplaced?
  • a)
    It may not be possible to selectively catch red tramfish.
  • b)
    The cost of extracting oil from the bones of the red tramfish is higher than the cost of artificially developing the same oil.
  • c)
    The oil extracted from the bones of a single red tramfish is sufficient to produce more than four weeks of medication for an arthritis patient.
  • d)
    At the levels at which the red tramfish is now being caught, the rate of reproduction of the fish is less than the rate of fishing.
  • e)
    The proportion of population that is most affected by arthritis is expected to stay the same in the near future.
Correct answer is option 'D'. Can you explain this answer?

Saumya Shah answered
Pre-Thinking
Conclusion Clarification
The question stem asks us to support the idea that the fear of the locals may not be misplaced. In other words, we are to support the belief that the locals may be right in fearing that the species will soon be extinct.
Pre-Thinking Approach
To strengthen the conclusion, let’s look at the logical structure, focusing on linkages 1 and 2.  We will try to come up with an assumption by understanding the condition under which the conclusion is falsified.  The assumption then will be that condition does not exist and the conclusion can be strengthened by supporting this or any other assumption.
Linkage#1 & 2RT is a rare species and there is a sudden interest in it.
  • Falsification Condition:  Yes, RT could be caught extensively to produce the oil.  But what if the rate at which it  reproduces is significantly higher than the rate at which it will be caught.  If this were true, then the conclusion will be falsified. Accordingly, a strengthener could say that:
  • Instance: The rate at which RT reproduces is not higher than the rate at which it is being currently caught.
In the above scenario, the fear of the locals seem valid as surely if fishing RT at the above rate carries on then it is possible that the species may go extinct.
Linkage # 1 - RT is a rare species that can be found only in some of the lake shores of the Silver Lakes region.
  • Strengthener : The fact that RT is a rare species and is naturally found only in a particular lake supports the locals’ fear. However, what if RT can be successfully bred in captivity? It could then be argued that their fear is not valid. Therefore, a possible strengthener could be to rule out this possibility.
  • Instance:  All attempts to breed RT in captivity have been unsuccessful so far and there is no breakthrough technology in sight that provides hope in the matter.
With this understanding in mind, let's take a look at the option statements.
Answer Choices
It may not be possible to selectively catch red tramfish.
Incorrect - Irrelevant
A
This option does not impact the conclusion in anyway as we do not know that whether people who have a sudden interest in RT will be interested in catching only RT or may prefer getting hold of other fish alongwith it.
B
The cost of extracting oil from the bones of the red tramfish is higher than the cost of artificially developing the same oil.
Incorrect - Irrelevant
Neither the choice nor the passage gives us any information to establish whether the cost of extracting oil is going to be a factor for the people interested in catching RT. Hence, the information here is completely irrelevant.
C
The oil extracted from the bones of a single red tramfish is sufficient to produce more than four weeks of medication for an arthritis patient.
Incorrect - Irrelevant
As we don’t know how the quantity of oil obtained from a red tramfish relates to its probability of going extinct, this option is not relevant.
D
At the levels at which the red tramfish is now being caught, the rate of reproduction of the fish is less than the rate of fishing.
Correct
This choice is written along the lines of our pre-thinking per Linkage #1 & 2.
E
The proportion of population that is most affected by arthritis is expected to stay the same in the near future.
Incorrect - Irrelevant
We cannot determine what impact the information given in this choice would have on the conclusion drawn by the locals. It doesn’t tell us whether the absolute number of people most affected by arthritis is expected to go up or not.

The feedback collection boxes installed at the reception of our hotels do not really serve their real purpose. As these boxes are in plain sight, many customers do not feel comfortable depositing the feedback forms in front of the hotel staff. Moreover, the hotel staff can easily access and destroy the feedback form deposited by a guest who intends to report a negative experience. Accordingly, the hotel should make individual phone calls to guests after they have completed their stay, giving them a fair opportunity to express their real experience of staying at the hotel.
Which of the following would it be most important to consider while evaluating the new proposed method of obtaining feedback?
  • a)
    Whether the staff at the hotels will be under pressure to provide better service, fearing that they may be reported in the feedback call
  • b)
    Whether the proposed feedback mechanism has been successful for other hotels catering to the same kind of customers
  • c)
    Whether there will be any cost implications for the hotels of executing the proposed change in the feedback mechanism
  • d)
    Whether there will be any significant increase in the number of guests due to the proposed way in the feedback mechanism
  • e)
    Whether the feedback calls will be conducted by the staff of the hotel that the guest will have stayed in
Correct answer is option 'E'. Can you explain this answer?

Ujwal Iyer answered
Importance of Feedback Call Conductors
When evaluating the proposed method of obtaining feedback through phone calls, the identity of the caller is crucial for several reasons:
Perception of Bias
- If the feedback calls are conducted by staff from the hotel where the guest stayed, it may lead to biased responses. Guests might feel pressured to provide positive feedback to avoid potential backlash or discomfort.
Trust and Honesty
- Guests are more likely to share their honest experiences if they believe the feedback is confidential and not linked to the hotel staff. This can lead to more valuable insights that can help improve services.
Effectiveness of Feedback
- If the hotel staff conducts the calls, the quality and candor of the feedback may diminish. This can impact the overall effectiveness of the feedback mechanism, making it less useful for addressing areas needing improvement.
Guest Comfort
- Ensuring that feedback is collected by an independent party can enhance guest comfort, encouraging them to share unfiltered opinions, particularly regarding negative experiences.
Implementation of Changes
- Understanding the feedback's authenticity is vital for the hotel to implement meaningful changes. If feedback is skewed due to staff involvement, necessary improvements may not be realized.
In summary, the success of the new feedback method hinges significantly on who conducts the calls, making option 'E' the most critical factor to consider.

While fossil fuels are rapidly being replaced by inexhaustible natural resources in order to conserve perishable resources, fossil fuels are still required to a certain extent to run power plants that generate alternative forms of energy. Some experts are of the opinion that fossils located in submarine regions should no longer be accessed, since such regions are difficult to reach, and undersea mining operations cost at least twice as much as subterranean mining does.
Which of the following, if true, mostly strongly supports the recommendation that submarine fossils should continue to be mined?
  • a)
    The amount of time taken to access submarine fossils is at least twice the amount required to access subterranean fossils.
  • b)
    The usability of fossils for the purpose of fuel generation is dependent on the chemical composition of the fossil, and some submarine fossils do not meet the stipulated requirements.
  • c)
    Some perishable resources are located above the surface level of the ground in areas such as caves and river banks and can be accessed far more easily than subterranean fossils can.
  • d)
    Each unit of energy generated for each submarine fossil is at least twice as much as the energy generated for each unit of subterranean fossil.
  • e)
    Most companies in the mining industry are insured against injuries caused to their employees while mining for fossils in areas where accidents are frequent.
Correct answer is option 'D'. Can you explain this answer?

Pre-Thinking
Conclusion Clarification
The argument presents the conclusion of some experts. They recommend that fossils located in submarine regions no longer be accessed. The question requires us to support the conclusion that submarine fossils should continue to be mined. As this recommendation is against the conclusion drawn by some experts, the question requires us to weaken this conclusion.
Pre-Thinking Approach
Let’s see how we can make the conclusion less believable.  To do so, we will look at the logical structure, focusing on linkage 1 and the conclusion.  Thus, the way to weaken the conclusion will be to break this linkage i.e. find a new piece of information that  suggests that the recommendation made in the conclusion should not be followed i.e. submarine fossils should continue to be mined.
Linkage#1Submarine regions are difficult to reach, and undersea mining operations cost at least twice as much as subterranean mining does.
  • Weakener – What if despite the higher cost, mining submarine fossils is extremely advantageous? In such a case, the conclusion drawn by some experts won’t hold.
    • Instance 1: For example, what if the fossils found undersea are of significantly higher quality than the SM fossil in terms of the output of fuel they generate?
With this understanding in mind, let’s take a look at the option statements.
 
Answer Choices
A
The amount of time taken to access submarine fossils is at least twice the amount required to access subterranean fossils.
Incorrect - Opposite
This option favors the recommendation by some experts by highlighting a negative aspect associated with submarine fossils. 
B
The usability of fossils for the purpose of fuel generation is dependent on the chemical composition of the fossil, and some submarine fossils do not meet the stipulated requirements.
Incorrect - Opposite
This option too highlights a negative aspect associated with some submarine fossils. This information supports the conclusion drawn by some experts.
C
Some perishable resources are located above the surface level of the ground in areas such as caves and river banks and can be accessed far more easily than subterranean fossils can.
Incorrect - Irrelevant
This choice presents a superfluous comparison between SM fossils and perishable resources located above ground. The argument is about SM and UM fossils.
D
Each unit of energy generated for each submarine fossil is at least twice as much as the energy generated for each unit of subterranean fossil.
Correct
This option is written along the lines of our pre-thinking linkage. The higher energy generated can compensate for the higher cost.
E
Most companies in the mining industry are insured against injuries caused to their employees while mining for fossils in areas where accidents are frequent.
Incorrect - Irrelevant
First of all, insurance against injuries isn’t linked to the argument in anyway, Second of all, this choice doesn’t specify which operations these companies are involved in – UM or SM.

In North America, of the people who buy cell phones, the percentage of people who bought phones from local manufacturers has decreased by five percent in the past one year. Since many local phone manufacturers in North America depend heavily on the local population for their customer base, this reduction is likely to have caused a noticeable negative economic effect on these manufacturers.
Which of the following, if true, most seriously weakens the argument?
  • a)
    In the past year, the total number of people buying new cell phones has increased considerably in North America.
  • b)
    Some people from Asia also buy cell phones from manufacturers in North America.
  • c)
    The cost price of cell phones has increased considerably, and so the selling price has also increased correspondingly.
  • d)
    Many new cell phone manufacturers have entered the market in North America.
  • e)
    Those who do not purchase from local manufacturers do not pay a high price for phones.
Correct answer is option 'A'. Can you explain this answer?

Niharika Sen answered
Pre-Thinking
Conclusion Clarification
N/A
Pre-Thinking Approach
Let’s see how we can make the conclusion less believable.  To do so, we will look at the logical structure, focusing on linkage 1 and the conclusion. We see, that the author has drawn a conclusion on the basis of some statistics. Thus, the way to weaken the conclusion will be to show that a statistical problem exists with the interpretation of the data used to draw the conclusion.  
Linkage#1
  • Weakener – Show that author’s interpretation of the data (proportion ? = absolute number ?) is not warranted. In what scenario can the proportion of the people decline but the absolute number remain constant or increase?
It’s possible if the overall population of people buying cellphones in North America has increased significantly.
    • Instance: For example, if the population increased from 10,000 to 20,000 and the PPLM declined from 15 to 10% (decline of 5%). The number of people buying phones from local manufacturers has actually increased from 1500 to 2000.
With this understanding in mind, let’s take a look at the option statements.
Answer Choices
A
In the past year, the total number of people buying new cell phones has increased considerably in North America.
Correct
This option is written along the lines of pre-thinking.
B
Some people from Asia also buy cell phones from manufacturers in North America.
Incorrect - Irrelevant
 
The argument focuses only on the phone manufacturers that depend heavily on the local population for their customer base.
 
C
The cost price of cell phones has increased considerably, and so the selling price has also increased correspondingly.
Incorrect - Irrelevant
The argument focuses on the impact of decline in PPLM on the economic condition of these manufacturers.
D
Many new cell phone manufacturers have entered the market in North America.
Incorrect - Irrelevant
The author’s conclusion is based on an interpretation of the drop in the proportion of people buying phones from local manufacturers. Now why, the drop has taken place, for e.g. due to increased competition from the new cell phone manufacturers (we are not told that they are local), is not important.
E
Those who do not purchase from local manufacturers do not pay a high price for phones.
Incorrect - Irrelevant
Again, like choice D, a possible reason behind the drop is not important.

MixIt, a company that manufactures smoothie-makers, has been manufacturing smoothie-makers as well as spare parts for its products. Since old smoothie-makers develop faults and spare parts are required to fix them, MixIt should continue manufacturing spare parts for its older models in the future.
Which of the following, if true, would best support the recommendation that the company should no longer manufacture spare parts for its models?
  • a)
    After prolonged use, some of the older models of Mixit smoothie-makers become irreparable.
  • b)
    Not many manufacturers of smoothie-makers produce spare parts for their older models.
  • c)
    It is becoming increasingly cheaper to replace a smoothie-maker with a new one than to get it repaired.
  • d)
    The profit margins in the sales of spare parts are significantly lower than the profit margins in the sales of smoothie-makers.
  • e)
    Excellent customer service is one of the necessary requirements for a company to be successful in the smoothie-maker market.
Correct answer is option 'C'. Can you explain this answer?

Niharika Sen answered
Pre-Thinking
Conclusion Clarification
The conclusion presents a recommendation based on a given situation. The question stem presents another recommendation that goes against this conclusion.
Pre-Thinking Approach
Let’s see how we can make the conclusion less believable.  To do so, we will look at the logical structure, focusing on linkage 1and the conclusion.  Thus the way to weaken the conclusion will be to weaken or break this linkage by finding a new piece of information that will cast a doubt on the recommendation made in the passage.
Linkage#1Old smoothie-makers develop faults and spare parts are required to fix them.
  • Weakener – What if for MixIt customers there is a better alternative than getting old SMs repaired?
    • Instance 1: New models of SMs that are cost efficient can sway people away from getting the old SMs repaired.
 
  • Weakener - What if MixIt can outsource the same quality of spare parts for its products at a cheaper price? In such a case, MixIt can cater to the demand of its customers while realizing cost savings.
With this understanding in mind, let’s take a look at the option statements.
Answer Choices
A
After prolonged use, some of the older models of Mixit smoothie-makers become irreparable.
Incorrect - Irrelevant
The argument makes a recommendation on the basis of a current requirement, which is the demand for spare-parts to fix old smoothie-makers. So even if some models cannot be repaired, the requirement for the spare-parts for the ones that can be is still valid. 
B
Not many manufacturers of smoothie-makers produce spare parts for their older models.
Incorrect - Irrelevant
This information doesn’t argue for or against the conclusion as the argument is specific to the business decision MixIt should make.
C
It is becoming increasingly cheaper to replace a smoothie-maker with a new one than to get it repaired.
Correct
This choice is written along the lines of our pre-thinking per Instance 1. If it’s becoming cheaper to replace a SM with a new model, it is quite possible that the demand for the spare parts to fix old models will decrease progressively.
D
The profit margins in the sales of spare parts are significantly lower than the profit margins in the sales of smoothie-makers.
Incorrect - Irrelevant
The profit margins in the sales of spare parts may be lower yet significant enough that the company should continue manufacturing spare parts for its older models.
E
Excellent customer service is one of the necessary requirements for a company to be successful in the smoothie-maker market.
Incorrect - Irrelevant
The argument makes no link between customer service and manufacturing of spare parts.

The debate over whether obesity is a disease grows as obesity rates and the cost of treating obesity-related conditions increase in the United States. Proponents stress that obesity is a disease because it is a result of genetics and biological factors, or illnesses, including hypothyroidism and Cushing's syndrome, disorders that cause weight gain. Opponents argue that obesity is not a disease because it is the result of a person's chosen lifestyle, eating habits, and environment. However, in this ever flaring debate on the categorization of obesity, both the sides ignore the fact that categorization of obesity in either way would not address the underlying problems causing it, problems such as poverty, the prevalence of unhealthy processed foods, etc. The nation and its people would be benefited if we focus more on tackling the underlying issues than on debating the categorization.
In the argument above, the two portions in boldface play which of the following roles?
  • a)
    The first describes a circumstance whose validity is questioned in the argument; the second raises that question.
  • b)
    The first is an observation that the argument as a whole seeks to explain; the second is a claim that explains one aspect of the observation.
  • c)
    The first is an observation, for which two possible explanations are presented in the argument; the second dismisses both of those explanations.
  • d)
    The first is an observation that is not disputed by the argument; the second is a belief that supports the conclusion of the argument.
  • e)
    The first provides a reason to doubt the conclusion of the argument; the second is the opinion that creates that doubt.
Correct answer is option 'D'. Can you explain this answer?

Niharika Sen answered
Argument Analysis
 
Pre-Thinking
After the passage analysis, we should be able to pre-think the correct answer choice quite comfortably. Let’s look at the two boldface portions once again w.r.t. the main conclusion of the argument.
Main conclusion: The nation and its people would be benefited if we channelize our energies more on tackling the underlying issues than on debating the categorization.
Bold-face portion 1 (BF1): The debate over whether obesity is a disease grows as obesity rates and the cost of treating obesity-related conditions increase in the United States
Bold-face portion 2 (BF2): categorization of obesity in either way would not address the underlying problems causing it
We know from the passage analysis that BF 1 states an observation made by author. It is stated in order to develop the argument.  BF2 is a part of author’s opinion. It is supporting the main conclusion by stating that it’s more important to focus our energies on tackling the root causes of obesity than on debating whether it is a disease or not. Since BF2 mentions that categorizing obesity is not going to help in addressing the causes behind it, it forms a strong basis for the main conclusion.
With this understanding in mind, let’s take a look at the option statements.
Answer Choices
A
The first describes a circumstance whose validity is questioned in the argument; the second raises that question.
Incorrect – Both parts wrong
BF1 does describe a circumstance but its validity is never questioned. The author acknowledges that the debate regarding obesity is indeed growing. BF2 states that it’s not very useful to indulge in the debate mentioned in BF1.
B
The first is an observation that the argument as a whole seeks to explain; the second is a claim that explains one aspect of the observation.
Incorrect – Both parts wrong
BF 1 is an observation but the main purpose of the argument is not to explain why the debate over obesity is growing.  This information has been used as background information to develop the argument further. BF2 could be considered a claim but it doesn’t explain an aspect of BF1, rather it challenges the usefulness of  the growing debate mentioned in BF1.
C
The first is an observation, for which two possible explanations are presented in the argument; the second dismisses both of those explanations.
Incorrect – Both parts wrong
As discussed above, BF1 is an observation; however argument doesn’t present any explanations of this observation. Since there are no two explanations offered, the description of BF2 in this choice is completely invalid.
D
The first is an observation that is not disputed by the argument; the second is a belief that supports the conclusion of the argument.
Correct
Both boldface portions have been identified along the lines of our pre-thinking.
E
The first provides a reason to doubt the conclusion of the argument; the second is the opinion that creates that doubt.
Incorrect – Both parts wrong
BF1 does not provide any reason to doubt the conclusion of the argument. In fact it is the conclusion of the argument that questions the relevance of the growing debate mentioned in BF 1. As noted in the prethinking, BF2 is used to support the (main) conclusion and not create any doubt on the same.

Chinese restaurants that offer free jasmine tea with meals do better business than similar Chinese restaurants that do not offer it for free. But since restaurants offering this free tea are more likely to offer food that doesn’t contain MSG, a substance that is blamed for causing severe headaches in some people after consuming Chinese food, it is the repeat business of patrons allergic to MSG that is responsible for higher business and not the attraction of free tea.
Which of the following, if true, most strengthens the argument above?
  • a)
    Although MSG is a taste enhancer, some people cannot make out the difference between the food prepared with and without it.
  • b)
    No study has been able to prove a conclusive relation between MSG and the associated headaches with consuming food that contains MSG.
  • c)
    A large majority of the customers of the Chinese restaurants offering free jasmine tea don’t end up requesting jasmine tea.
  • d)
    Most restaurants that do not offer free jasmine tea end up not charging less than the usual price for the same when it is ordered by patrons.
  • e)
    Some customers at Chinese restaurants that do not offer free jasmine tea complain of severe headaches right after having a meal at those restaurants.
Correct answer is option 'C'. Can you explain this answer?

Explanation:

C. A large majority of the customers of the Chinese restaurants offering free jasmine tea don't end up requesting jasmine tea.
- This statement strengthens the argument by suggesting that the free jasmine tea may not be the main attraction for customers. If most customers do not end up requesting the free tea, then it is likely that the absence of MSG in the food is the main reason for repeat business.
- It implies that customers are not coming back just for the free tea but for other reasons such as the quality of food or the absence of MSG.
Therefore, option C provides additional support for the argument that it is the absence of MSG in the food that is responsible for higher business at Chinese restaurants offering free jasmine tea, rather than the free tea itself.

In Varisland, when a subscriber of one telecom operator calls a subscriber of a different telecom operator, the latter’s telecom company charges a small connection fee to the originating operator, a fee stipulated by the regulator and same for every operator.  Almost all the new telecom operators are enticing customers by charging much lower call rates than charged by the existing operators. Since the connection fee is one of the sources of revenue for new companies, the regulator should increase the connection fee, improving the financials of the new telecom operators and thereby enabling them to better compete in the market.
Which of the following statements provides the strongest reason to suggest that the recommendation of the author would have an effect opposite to the one envisaged? 
  • a)
    The connection fee is such a small component of revenues of any telecom companies that no company could survive by just relying on this.
  • b)
    In the past, whenever regulator has been advised a course of action by an outside agency, the regulator has taken a completely opposite action step.
  • c)
    The financial health of a telecom company is determined by the total profits generated by the company, irrespective of the source of revenues.
  • d)
    Due to highly competitive call rates, the number of outgoing calls from an average user of a new telecom company is expected to be much greater than the number of incoming calls.
  • e)
    Increase in the connection fee will not be welcomed by many existing telecom operators as they will have to pass on the charges levied on them by the new operators to their customers.
Correct answer is option 'D'. Can you explain this answer?

Understanding the Argument
The author's recommendation suggests that increasing the connection fee will improve the financial health of new telecom operators in Varisland, enabling them to compete better in the market. However, this may not be the case.
Why Option D is Strongest
The strongest reason against the author's recommendation is option D, which states that due to highly competitive call rates, the number of outgoing calls from an average user of a new telecom company is expected to be much greater than the number of incoming calls.
Key Points to Consider:
- Revenue Dynamics: If new telecom operators are charging lower call rates, they are likely to attract more customers who will make numerous outgoing calls.
- Connection Fee Impact: The connection fee is only charged for incoming calls, which means that if outgoing calls from new operators are significantly higher, they will not benefit much from the increased connection fee.
- Market Inequality: As users of new telecom operators make more outgoing calls, the originating operators may not generate sufficient incoming call revenues to offset the increased connection fee.
- Long-term Viability: If new operators cannot rely on incoming calls to benefit from the connection fee, the intended effect of improving their financials may backfire, making it even harder for them to compete.
Conclusion
Thus, increasing the connection fee may inadvertently weaken the financial position of new telecom operators instead of strengthening it, contradicting the author's intended outcome. This highlights the complexity of market dynamics where revenue sources must be considered holistically.

Parent: It is as misleading to conclude that decreasing attendance in after-school extra classes shows that our children’s academic requirements are finally being fully met in regularly-scheduled classes as it is to reason that teachers do not get negative ratings from students because students respect and admire teachers. The real cause behind the decreasing attendance is that the same teachers who teach the regular classes teach the after-school ones, and students don’t like the methodology used by these teachers in the first place. 
The parent’s statements, if true, best support which of the following conclusions?
  • a)
    The reason teachers do not get a negative rating is that the students are scared about the repercussions their votes could have on their academic assessments.
  • b)
    Teaching methodologies across schools need to be evaluated to best cater to the students’ needs and requirements.
  • c)
    The management of the school wants to cover up for their lack of an efficient teaching methodology by implementing the rating system.
  • d)
    The students’ academic needs are not being fulfilled in the regular classes.
  • e)
    If the school changes its teaching methodology in regular classes, most of the students will be motivated to attend the free after-school extra classes 
Correct answer is option 'E'. Can you explain this answer?

A is irrelevant
B says fact which is half told in the passage and a suggestion, but we need strengther.
C again irrelevant.
D this might be the case but it doesn't strengthen the argument of parents.
E says if method changes in regular then students will attend extra classes.bkz we are strengthening the logic that SINCE STUDENTS DON'T LIKE METHODS OF REGULAR CLASS TEACHER,WHAT IS THE MEANING OF JOINING EXTRA CLASS.

So E is clear cut winner.

Triumba birds reside in the polar regions and need to migrate to warmer areas in winters since winters in the polar regions are not conducive to the survival of the birds. In recent years, because of some unanticipated genetic changes, the flying capability of the birds has been declining and is expected to continue do so in the future. 
Which of the following statements is best supported by the passage above?
  • a)
    At least one species that resides in the polar regions cannot survive in the summer by migrating to other regions.
  • b)
    If Triumba birds don’t develop measures to counter adversarial genetic changes, they face a likelihood of extinction.
  • c)
    Over the past few years, some of the Triumba birds would not have flown back to the polar regions because of limited flying ability.
  • d)
    Unless Triumba birds develop an ability to live in the polar regions during winter, they will be extinct in some time.
  • e)
    The flying capability of Triumba birds is worse than that of other polar region birds that do not migrate to warmer areas in the winter.
Correct answer is option 'B'. Can you explain this answer?

Sandeep Mehra answered
Pre-Thinking
Conclusion Clarification
The question stem says that “Which of the following statements is best supported by the passage above?”. The question requires us to draw a conclusion based on statements in the passage. Please note that the conclusion must be based only on the information provided in the passage.
Pre-Thinking Approach
In order to see what can be concluded from the argument, we need to observe all the facts given in the argument and see if the information in the passage is true, what must also be true.
Let us combine two pieces of information presented in the passage:
  1. Triumba birds need to migrate to warmer areas in winters since winters in the polar regions are not conducive to the survival of the birds.
  2. The flying capability of the Triumba birds is expected to continue to decrease in the future.
Understanding the situation: So we know that the birds have to migrate to other areas in winters because if they don’t do so, their survival is in danger. Combining this information with the fact that their flying capability is declining and is likely to keep declining in the future, we can see that the survival of the birds is in danger unless there is a positive development in their flying capability. Therefore, we can conclude that:
  • The Triumba birds are likely to become extinct unless they develop something that enables them to counter the adverse impact of the genetic changes so that they remain capable of flying to warmer places during winter.
With this understanding in mind, let's move to the option statements.
Answer Choices
A
At least one species that resides in the polar regions cannot survive in the summer by migrating to other regions.
Incorrect - No information given 
This option talks about surviving the summers in the polar region. However, the argument only talks about the survival of Triumba birds in these regions during winter
B
If Triumba birds don’t develop measures to counter adversarial genetic changes, they face a likelihood of extinction.
Correct
This option says that it is likely that the birds will become extinct unless they are able to counter the genetic changes that are affecting their flying capability.It matches our prethinking analysis as it talks about an outcome that is dependent on a positive impact in the flying ability of the birds.
C
Over the past few years, some of the Triumba birds would not have flown back to the polar regions because of limited flying ability.
Incorrect - No information given
This option suggests that some Triumba birds may have already become incapable of flying back to the polar regions. However, the passage does not suggest that the decline in the flying capability is at a stage where some of these birds have or had become incapable of flying.  
D
Unless Triumba birds develop an ability to live in the polar regions during winter, they will be extinct in some time.
Incorrect - No information given
Assuming that the decline in the capability of the birds will definitely carry on, this choice talks about a certainty about the future of these birds. However, as noted in the pre-thinking, the passage talks about a likelihood (expected to)of a further decrease in their flying capability. Also, the option does not consider that the birds might still avoid extinction by countering the adverse effect of the genetic changes. 
E
The flying capability of Triumba birds is worse than that of other polar region birds that do not migrate to warmer areas in the winter.
Incorrect - No information given
The option statement presents an irrelevant comparison between the flying ability of the Triumba birds with that of other polar birds. The passage gives us no information about the flying ability of other birds. 

New research shows that some marine organisms such as eels are facing a severe threat to their survival because of the increasing marine noise pollution. Researchers found that when exposed to acute noises such as those typical of large fishing ships, European eels reacted very differently to predator attacks; the likelihood of their response to an ambush by a predator decreased by 50% and the reaction times of the ones that did respond were delayed by as much as 25%.
Which of the following can be concluded on the basis of the information given in the above passage?
  • a)
    In the presence of acute sounds such as those coming from a large fishing ship, marine organisms such as eels cannot hear predators approaching.
  • b)
    Fishing ships contribute to the endangerment of many marine organisms both directly and indirectly.
  • c)
    For European eels, the likelihood of a delay in their reaction time to an ambush by a predator increases with the increase in noise pollution.
  • d)
    Marine noise pollution increases the likelihood of a marine predator successfully catching its prey in some instances.
  • e)
    When exposed to acute noises in a danger situation, some marine organisms lose control of most of their reflex actions.
Correct answer is option 'C'. Can you explain this answer?

Pre-Thinking
Conclusion Clarification
The conclusion states that the reduction in the prices of staple foods that are given to authorized dealers by government will significantly improve the economic conditions of the people from LIG, thus achieving the aim of the government.
Pre-Thinking Approach
In order to evaluate the linkage, we’ll pre think assumptions by thinking of conditions that will falsify the conclusion. The assumptions then will be that these conditions don’t exist. To do so, let’s look at linkage 1 and understand the conditions under which it doesn’t hold. 
Linkage#1In an attempt to improve the economic and social welfare of the people, the government has recently announced a scheme under which it will provide certain staple foods at 30% discount to the authorized dealers.
  • Falsification Condition:  What if, people from LIG spend a negligible proportion of their income on buying staples that will be discounted under the scheme? In such a case, these people could only be benefitted, if at all, to a small extent and thus the conclusion that price reduction on such staples would significantly improve the economic condition of people from LIG would not be valid. Clearly, in such a case, the linkage will not hold.
  • Assumption1: A significant share of the income of people from LIG is currently spent on buying staple foods that would be discounted under the scheme.
 
  • Falsification Condition:  What if, the prices that ADs charge for the staple foods do not reflect the lower prices they would be paying the government, i.e. what if ADs continue to charge the same price from the people despite the discount provided to them by the government? In such a scenario, the people from LIG will not benefit from the scheme and thus the linkage would not hold.
  • Assumption 2: The prices that dealers charge for the staple foods will reflect the discount they will receive from the government.
An answer choice that evaluates any of the fundamental assumptions made by the author will be the correct choice. With this pre-thinking in mind, let’s evaluate the answer choices.
Answer Choices
A
Whether discounting the prices of staple foods is the only way to improve the economic and social welfare of the people
Incorrect – Irrelevant
The argument requires us to evaluate whether discounting the prices of staple foods will improve the economic and social welfare of the people. Whether there are other ways that may lead to the same effect is not relevant to the argument.
B
Whether people with medium or high income groups will also benefit from the scheme
Incorrect – Irrelevant
The argument focuses only on evaluating the impact of the scheme on people from LIG and thus it’s not relevant to know whether other groups will also benefit from the scheme.
C
Whether the expenditure on staple foods under question forms a significant chunk of the income of people from low income groups
Correct
This choice is written along the lines of our pre-thinking per Linkage assumption1. If the answer to the question is yes, the argument is strengthened. Otherwise, it is weakened.
D
Whether the people from low-income groups will appreciate some other kind of financial assistance more than the one being offered by the government
Incorrect – Irrelevant
The fact that people will prefer some other kind of assistance doesn’t help us evaluate about the one offered by the government. Thus, people’s preference regarding the kind of assistance from the government is immaterial here.
E
Whether this scheme would have an adverse impact on the budgetary allocation of other welfare initiatives taken by the government.
Incorrect – Irrelevant
Impact of the scheme on other initiatives is not relevant to the argument.  We are only concerned with the impact of the scheme on economic condition of the people from LIG.
 

The seafront properties in Southern coastal regions of India had always been expensive and most sought after. After the Tsunami hit the coastal regions of India last year, the prices of the seafront properties dropped as most inhabitants started evacuating their homes and moved inlands.  As a result the property values of inland homes started increasing.  The installation of the Tsunami sensors will definitely bring the seafront property prices to its original glory and thus the original price.
Which of the following, if true, most calls into question the reasoning on which the plan is based?
  • a)
    Many good schools were also asked to open their branches near to the seashore to further increase the interest of people in buying properties near the sea .
  • b)
    Because of the economic slowdown in India, the cost of all properties have dropped at least 10% compared to last year and the trend is expected to continue this year.
  • c)
    Many not-for-profit organizations have been requesting the installation of Tsunami sensors for a very long time.
  • d)
    Some people have always preferred to buy properties inland rather than on seafronts.
  • e)
    Since this was the first time people in India experienced a Tsunami, many owners of the seafront properties did not feel safe to return to their properties before the sensors were installed.
Correct answer is option 'E'. Can you explain this answer?

This qn is not is an official qn . don't mind but this is not a valid qn atleast on GMAT . option E is acting as strengthener.rather B is a better option.but my suggestion is don't go for unofficial gnat questions.

In an effort to promote the culture of movie going, the media industry of Mumbai has decided to not release DVDs in India till about 5 weeks after the movie has been released. The industry director argues that this would force those individuals who are financially well off and wait eagerly for the movie to be released but are too lazy to go to a cinema hall to visit the cinema halls more often.
Which of the following if true indicates that the plan to hold off the release of DVD's of a movie is ill designed to have the desired effect?
  • a)
    Since the cost of watching movies in a movie theater is much higher than that of watching it on the DVD, only a few people who prefer to watch the movie on DVD would go to the movie theater.
  • b)
    If released 5 weeks late, the movie industry may have to reduce the price of the DVD to make it attractive for people to purchase the same.
  • c)
    A recent survey has indicated that 15% of the people may forgo watching a movie if the movie is not immediately released on the DV
  • d)
    D. Even if the DVDs are not released in India, there is a high probability that DVDs from neighboring countries will make their way to the country.
  • e)
    Movie theaters in a number of metropolitan cities are running at 70% capacity and may have to increase their capacity to accommodate the surge in demand.
Correct answer is option 'D'. Can you explain this answer?

Niharika Sen answered
Pre-Thinking
1. Assumptions:
  • Plan will be implemented properly.  For example, there will not pirated DVDs available.
  • There is increased seating capability in Cinema halls to accommodate increased inflow of people.
2. Weakeners:
  • Evidence suggesting that such people will be able to get their hands on DVDs through alternate channels.
  • Evidence suggesting that there is no capacity available for target population.
Answer Choices
A
Since the cost of watching movies in a movie theater is much higher than that of watching it on the DVD, only a few people who prefer to watch the movie on DVD would go to the movie theater.
The passage specifically talks about financially well off people.  Thus, this choice does not weaken the argument.
B
If released 5 weeks late, the movie industry may have to reduce the price of the DVD to make it attractive for people to purchase the same.
The passage is not concerned about whether the DVDs are attractive to people to purchase.  The passage is concerned about making people visit the cinema halls.
C
A recent survey has indicated that 15% of the people may forgo watching a movie if the movie is not immediately released on the DVD.
This choice is again concerned with people watching movie on DVD. It does not have any impact on whether this movie if not released in the DVDs will result in target audience going to cinema halls.
D
Even if the DVDs are not released in India, there is a high probability that DVDs from neighboring countries will make their way to the country.
This choice states that the plan would not be implemented properly because eventually people will get their hands on the DVDs.  This choice thus effectively weakens the argument.
E
Movie theaters in a number of metropolitan cities are running at 70% capacity and may have to increase their capacity to accommodate the surge in demand.
This choice supports the conclusion because it states that the plan will be successful and the movie theatres may need to increase their capacity.

In Freud’s system of psychoanalysis, the human brain consists of three parts: the id, which is the part driven by instincts; the super-ego, which is the moral center of the brain; and the ego, which is the pragmatic or realistic part. Unlike the neurosciences, which categorize the brain into somatic units that correspond to physical areas in the brain, the Freudian system divides the brain into categories that exist solely at the symbolic or conceptual level.
The considerations above best serve as part of an argument that
  • a)
    somatic categories are more scientifically credible than symbolic categories.
  • b)
    the neurosciences reject Freud’s categories as baseless.
  • c)
    the Freudian system of psychoanalysis is not a neuroscience.
  • d)
    unlike Freud, neuroscientists do not believe that the brain has a somatic moral center
  • e)
    science in Freud’s time was not developed enough to identify the brain’s somatic structure.
Correct answer is option 'C'. Can you explain this answer?

Sahana Mehta answered
The Freudian System of Psychoanalysis
The Freudian system offers a conceptual framework for understanding human behavior that is fundamentally different from the neurosciences.
Key Components of Freud's Theory
- Id: Represents instinctual drives and desires.
- Super-ego: Functions as the moral compass, guiding ethical behavior.
- Ego: Balances the demands of the id and super-ego with the realities of the external world.
Conceptual vs. Somatic Understanding
- Freud’s divisions are symbolic and do not correspond to specific physical areas in the brain.
- This system provides insight into psychological conflicts and motivations rather than physiological functions.
Why Option C is Correct
- The statement highlights that Freud's psychoanalytic framework is not rooted in neuroscience, which relies on empirical and biological evidence.
- The distinction between symbolic and somatic categories emphasizes that Freud's categories exist in a conceptual realm, separate from the scientific study of brain structures.
Implications of the Distinction
- Understanding this difference is crucial for recognizing the limitations of psychoanalysis when compared to neuroscientific approaches.
- It clarifies that while both aim to explain human behavior, their methodologies and foundations are fundamentally different.
In summary, option C effectively encapsulates the essence of Freud's psychoanalysis as a non-neuroscientific approach, focusing on symbolic categories rather than physical brain structures.

Prior to 1971, the United States was on various forms of a gold standard through which the value of the dollar was backed by gold reserves and paper money could be redeemed for gold on demand. Since 1971, the United States dollar has been a fiat currency backed by "full faith and credit” of the government and not backed by, valued in, or convertible into gold. A gold standard restricts the ability of government to print money at will and to spend more than it earns in tax revenue. Besides, the economy has historically performed best under a gold standard. Therefore, for reasons above, the voters should vote for a party that backs the return of the gold standard in the country.
Which of the following data sets can be best used by the author of the above argument to further his conclusion?
  • a)
    Major economic indicators of United States since 1980.
  • b)
    Expenditure on education and healthcare from 1960 to 2000.
  • c)
    Money printed by the government as proportion of the Gross Domestic Product from 1950 to 1970.
  • d)
    Growth in the difference between the revenue and expenditure figures for the government for the last 100 years beginning from 1901.
  • e)
    Economic growth in the country in 1971 and in 1972.
Correct answer is option 'D'. Can you explain this answer?

Devansh Chawla answered
Data Set to Support the Return of Gold Standard Argument

Introduction:
The given argument talks about the benefits of returning to a gold standard in the United States. To support this argument, the author needs to provide data that shows how the economy has performed historically and how it has changed since the abandonment of the gold standard in 1971.

Data set:
The best data set to support the return of the gold standard argument would be the growth in the difference between the revenue and expenditure figures for the government for the last 100 years beginning from 1901. This data set provides a historical perspective on how the government has managed its finances and how it has affected the economy.

Heading: Historical Data on Government Finances

- The growth in the difference between the revenue and expenditure figures for the government is an important indicator of how well the government is managing its finances.
- This data set provides a historical perspective on how the government has managed its finances and how it has affected the economy.
- The data for the last 100 years beginning from 1901 shows that the government has consistently spent more than it has earned in tax revenue.

Heading: Impact of Abandoning Gold Standard

- The abandonment of the gold standard in 1971 allowed the government to print money at will and spend more than it earns in tax revenue.
- This led to a significant increase in the difference between the revenue and expenditure figures for the government, which has continued to grow over the years.
- This has also led to a decrease in the value of the dollar and an increase in inflation.

Heading: Benefits of Return to Gold Standard

- A return to the gold standard would restrict the ability of the government to print money at will and spend more than it earns in tax revenue.
- This would lead to a decrease in the difference between the revenue and expenditure figures for the government and a more stable economy.
- Historical data shows that the economy has performed best under a gold standard.

Conclusion:
In conclusion, the growth in the difference between the revenue and expenditure figures for the government for the last 100 years beginning from 1901 is the best data set to support the return of the gold standard argument. This data set provides a historical perspective on how the government has managed its finances and how it has affected the economy. The data shows that the government has consistently spent more than it has earned in tax revenue, and the abandonment of the gold standard in 1971 has allowed it to print money at will and spend more than it earns. A return to the gold standard would restrict the ability of the government to print money at will and spend more than it earns in tax revenue, leading to a more stable economy.

Studies have shown that the number of times a car breaks down per year is significantly dependent on whether the owner conducts regular maintenance for the vehicle. Most customers who buy luxury cars are extremely particular about the maintenance of the cars. The owners of luxury cars who employ chauffeurs are more careful in the maintenance of their cars than the owners of economy cars are.  Therefore, such luxury car owners will suffer fewer car breakdowns than the owners of economy cars.
Which of the following, if true, most strengthens the conclusion drawn above? 
  • a)
    On average, luxury cars driven by chauffeurs have almost the same number of breakdowns that sports cars driven by car owners do.
  • b)
    Most chauffeurs employed by luxury car owners are trained to fix breakdowns.
  • c)
    Only luxury cars come with a special training program for the drivers of the cars, a program focused on the careful handling of these cars.
  • d)
    On average, the cost per breakdown in a luxury car is not higher than the cost per breakdown in an economy car.
  • e)
    Many luxury cars give the same mileage per unit of fuel that most economy cars do.
Correct answer is option 'C'. Can you explain this answer?

Maya Khanna answered
Pre-Thinking
Conclusion Clarification
N/A
Pre-Thinking Approach
To pre-think assumptions, let’s see how we can falsify this conclusion.  To do so, we will look at the logical structure, focusing on linkage 1.  We will understand the condition under which the conclusion is falsified.  The assumption then will be that condition does not exist and any choice that states one of the assumptions or supports it will be a strengthener.
Linkage#1
  • Falsification Condition:  Yes, the car breakdowns are dependent on maintenance and LC owners are more careful about maintenance than EC owners. But what if LCs are inherently significantly more fragile than ECs? In that case, even if they are maintained better, they could still suffer more breakdowns because of being less sturdy. If this were true, then the conclusion will be falsified.
  • Assumption: Luxury cars are not prone to significantly more breakdowns in general than economy cars.
Linkage#2 –Falsification Condition:  Yes, the LC car owners who employ chauffeurs are more careful about the maintenance of their cars but what if their drivers are not careful about the handling of the car? In that case, if the LCs are being driven by the drivers for a significant proportion of time then they could end up in a condition worse than ECs. If this were true, then the conclusion will be falsified. Accordingly, a possible strengthener could be:
  • The chauffeurs employed by the owners of LCs are careful about the handling and maintenance of these cars.
With this understanding in mind, let's take a look at the option statements.
Answer Choices
A
On average, luxury cars driven by chauffeurs have almost the same number of breakdowns that sports cars driven by car owners do.
Incorrect - Irrelevant
This choice compares luxury cars driven by chauffeurs with an irrelevant section.
B
Most chauffeurs employed by luxury car owners are trained to fix breakdowns.
Incorrect - Irrelevant
The conclusion is concerned with the chances of the occurrence of an event whereas this option statement talks about the situation when the event has already occurred.
C
Only luxury cars come with a special training program for the drivers of the cars, a program focused on the careful handling of these cars.
Correct
This choice is written along the lines of our pre-thinking per Linkage #2. It tells us that these drivers are possibly trained regarding the careful handling of these cars, a factor that could contribute to fewer breakdowns in such cars.
D
On average, the cost per breakdown in a luxury car is not higher than the cost per breakdown in an economy car.
Incorrect - Irrelevant
This option is not relevantas it talks about cost of breakdowns, which the argument is not concerned with.
E
Many luxury cars give the same mileage per unit of fuel that most economy cars do.
Incorrect - Irrelevant
This option is not relevant as it talks about mileage per unit of fuel, which the argument is not concerned with.

Since the last few years, bird watching has become increasingly popular as a hobby in Ohio. The National Parks in Ohio have also been conducting bird watch programs for the past decade. Certainly, the increased interest in bird watching among the Ohio residents is because of the bird watch programs conducted by Ohio State National Parks.
Each of the following, if true, provides some support for the historians’ account described above EXCEPT:
  • a)
    Ohio State has widely publicized the Bird Watch programs conducted by the National Parks.
  • b)
    Ohio state is the house of many exotic and beautiful birds.
  • c)
    The bird watch programs are being conducted regularly by most of the National Parks in Ohio.
  • d)
    As part of the Bird Watch programs, the National Parks also provide good materials and help the bird watchers identify the birds.
  • e)
    Before the National Parks started conducting Bird watch programs, not many people in the state of Ohio were interested in bird watching.
Correct answer is option 'B'. Can you explain this answer?

Palak Yadav answered
Understanding the Context
The question revolves around the growing popularity of bird watching in Ohio and its potential correlation with the bird watch programs conducted by the National Parks in the state. The aim is to identify which statement does not support the claim that these programs have driven interest in bird watching.
Analysis of Options
- Option A: Ohio State has widely publicized the Bird Watch programs conducted by the National Parks.
This supports the claim by suggesting that increased awareness could lead to more participants in bird watching.
- Option B: Ohio State is the house of many exotic and beautiful birds.
While this may make Ohio a desirable location for bird watchers, it does not directly link the increase in interest to the National Parks' programs. Hence, this option does not support the claim.
- Option C: The bird watch programs are being conducted regularly by most of the National Parks in Ohio.
This indicates accessibility and opportunity for residents to engage in bird watching, supporting the idea that programs have driven interest.
- Option D: As part of the Bird Watch programs, the National Parks also provide good materials and help the bird watchers identify the birds.
This enhances the attractiveness of the programs and encourages participation, thus supporting the claim.
- Option E: Before the National Parks started conducting Bird watch programs, not many people in the state of Ohio were interested in bird watching.
This directly supports the claim by indicating a shift in interest due to the programs.
Conclusion
In summary, while options A, C, D, and E provide supporting evidence for the increase in bird watching interest due to the National Parks' programs, option B does not establish a direct link to the programs, making it the correct answer.

Since the quality of a product is defined by the features and the lifetime it has, food processors produced by Japanese manufacturers are not, on an average, of a superior quality to those produced by Chinese manufacturers. The Japanese food processors offer similar features in comparative price ranges, and they have the same average life-time as the Chinese ones.
Which of the following, if true, most supports the assertion that Japanese food processors may be of a superior quality to the Chinese ones?
  • a)
    The Chinese food processors are comparatively easier to use than the Japanese ones.
  • b)
    A survey conducted by the independent trade magazine Business Now showed that on average Japanese manufacturers have more service-centers for their products than their Chinese counterparts.
  • c)
    Chinese food processors have a comparatively higher malfunction rate than that of Japanese food processors, and studies have proven that a lower malfunction rate is correlated with efficient functioning of the original features offered.
  • d)
    Both the Japanese and the Chinese food processors use the FP-1000 technology, which is known to require constant servicing.
  • e)
    The features of Japanese food processors that are in a higher price range than most Chinese food processors are far more advanced than those of Chinese food processors.
Correct answer is option 'C'. Can you explain this answer?

Understanding the Assertion
The assertion states that Japanese food processors are not of superior quality compared to Chinese ones, as they have similar features, price ranges, and average lifetimes. To argue that Japanese food processors may indeed be of superior quality, we need supporting evidence that highlights a difference in performance or reliability.
Why Option C Supports the Assertion
Option C states that Chinese food processors have a higher malfunction rate than Japanese ones and that a lower malfunction rate correlates with efficient functioning of the original features offered. This statement provides the following key insights:
- Malfunction Rate: A higher malfunction rate in Chinese food processors indicates that they are less reliable than their Japanese counterparts.
- Correlation with Functionality: The assertion that lower malfunction rates correlate with efficient functioning implies that Japanese food processors not only function effectively but also maintain their functional features over time, enhancing their perceived quality.
- Quality Perception: If Japanese food processors are less prone to malfunctions, consumers may view them as superior, even if their features and lifetimes are similar to those of Chinese processors.
Conclusion
Overall, option C provides crucial evidence that distinguishes Japanese food processors from Chinese ones, supporting the claim of superior quality through reliability and functionality. This difference in malfunction rates can be a significant factor for consumers when assessing the overall quality of the products.

Unless more teachers are hired soon, the student-teacher ratio in schools is likely to increase. Such an increase is likely to interfere with Green Earth School’s declaration that it will always maintain a standard of quality in its teaching. So, the school’s management is considering hiring more teachers. 
Which of the following would be the most important for the school’s management to consider before deciding on hiring more teachers?
  • a)
    Whether the new people who are hired as teachers would otherwise be hired at other competing schools
  • b)
    Whether the curriculum presently followed by the school matches the standards of quality that the management wants to set
  • c)
    Whether the need to hire new teachers would lead to the selection of applicants who lack the qualifications and experience required to meet the existing standards
  • d)
    Whether the parents of most students would be willing to compromise on the standards guaranteed by the school
  • e)
    Whether the school would be able to take in a greater number of students if more teachers are hired
Correct answer is option 'C'. Can you explain this answer?

Amrutha Jain answered
Pre-Thinking
Conclusion Clarification
Per se, there is no main conclusion given in the argument. The argument gives us a situation and underlying reason for the same. However, the question stem takes the situation one step further and says: “Which of the following would be the most important for the school’s management to consider before deciding on hiring more teachers?” Therefore, we need to identify a question that can help the management take a firm decision on hiring more teachers. Hence the conclusion, for the purpose of evaluation, becomes: hiring more teachers is consistent with always maintaining a standard quality of teaching.
Pre-Thinking Approach
In order to evaluate the argument, we’ll pre-think assumptions and ask questions that can tell us whether they are correct or not. To do so, let’s see how we can falsify the conclusion.  Let’s look at the logical structure, focusing on linkage 1.  We will understand the conditions under which the conclusion is falsified.  The assumption then will be that these conditions don’t exist. 
Linkage#1
Falsification Condition:  A scenario that shows that hiring more teachers will not help maintain the standard quality of GES’s teaching.
  • Instance: The new hires might not be capable of maintaining the standard of quality in GES’s teaching.
    • Assumption: The new hires will be capable of maintaining the standard of quality in GES’s teaching.
Any question that helps us evaluate the fundamental assumption made by the author will be the correct answer.
With this understanding in mind, let’s take a look at the answer choices.
Answer Choices
A
Whether the new people who are hired as teachers would otherwise be hired at other competing schools
Incorrect - Irrelevant
The argument focuses on the decision to hire new teachers to address the student – teacher ratio. Whether the teachers would be hired by competitors or not if hiring doesn’t happen is irrelevant.
B
Whether the curriculum presently followed by the school matches the standards of quality that the management wants to set
Incorrect - Irrelevant
The argument focuses on the maintenance of standard of quality by maintaining student – teacher ratio. Therefore, curriculum quality is of no relevance.
C
Whether the need to hire new teachers would lead to the selection of applicants who lack the qualifications and experience required to meet the existing standards
Correct
This option questions the validity of the argument by questioning the assumption that we identified in our pre-thinking analysis. 
D
Whether the parents of most students would be willing to compromise on the standards guaranteed by the school
Incorrect - Irrelevant
The argument talks about the management’s actions to adhere to the quality in teaching. Therefore, parents’ willingness to compromise is of no relevance.
E
Whether the school would be able to take in a greater number of students if more teachers are hired
Incorrect - Irrelevant
The option does not address the quality maintenance in teaching. 

Some geologist propose haloclasty, a process of salt crystallization responsible for the weathering of rocks, as an explanation for the specific weathering pattern observed on the Egyptian Sphinx. This explanation is rejected by some other geologists who hold the opinion that the type of weathering evident on the Sphinx enclosure walls, along with that on the core blocks of the Sphinx and Valley temples, could have only been caused by prolonged and extensive rainfall and hence date the construction of the Sphinx back to the 6th or 5th millennium BC.Which of the following, if true, provides the best support for the assertion that the weathering on the Sphinx could not have been due to haloclasty but instead due to extensive rainfall in the area?
a)The geologists who oppose the haloclasty theory for the weathering noticed on the Sphinx accept that it may be responsible for the weathering noticed at many places on the Giza plateau.
b)Many geologists have concluded that haloclasty does not account for the absence of a similar weathering on other rock surfaces in the Giza pyramid complex, which were cut from the same limestone beds as the Sphinx.
c)Certain other studies have shown that the erosion observed on the sphinx was indeed driven by moisture derived from atmospheric precipitation such as dew, which ultimately led to the process of haloclasty.
d).Many geologists agree that the peculiar horizontal bands observed on the Sphinx were not caused by haloclasty but by wind erosion.
e)The weathering caused by haloclasty should have operated evenly on all exposed limestone surfaces of the Sphinx whereas the erosion observed indicates that the weathering is particularly evident on surfaces that would have been especially vulnerable to heavy downpour
Correct answer is option 'E'. Can you explain this answer?

The correct answer to this question should be option D, as in GMAT we give preference to those answers in which the comparison with respect to all the parties involved are mentioned. Option D is a clear case of such an answer. Moreover option E talks about wind erosion which is completely irrelevant.

In an effort to prevent accidents from distractions caused by texting while driving, auto companies are now offering built- in audio systems that link themselves through the Bluetooth technology to the individual’s mobile phone and automatically read texts as and when they are received. These companies are hopeful that their initiative will be appreciated by people who are increasingly voicing concerns over the rising number of auto-accidents reported in which the cause of the accident was the driver’s lack of focus on the road while texting. 
Which of the following, if true, most strongly supports the assertion that the people will not appreciate the initiative taken by the auto companies?
  • a)
    The price of automobiles fitted with such audio-systems is expected to be higher than the price of automobiles not fitted with such systems.
  • b)
    Consumers are generally apprehensive of any major alteration in the standard features offered in an automobile.
  • c)
    Studies have shown that five seconds is the average time a driver’s eyes are off the road while texting.
  • d)
    In the past five years, most innovations in the auto-industry have received only a lukewarm response from industry experts.
  • e)
    It is a commonly known fact that an individual driving a vehicle can lose focus by either a lapse in visual attention or an audio distraction.
Correct answer is option 'E'. Can you explain this answer?

Pre-Thinking
Conclusion Clarification
There is no stated conclusion in the passage. However, the question stem asks us to support an assertion that goes against the expectation of auto companies.  Hence, we are asked to weaken this expectation, which we’ll treat as the conclusion for the purpose of this question.
Pre-Thinking Approach
Let’s see how we can make the conclusion less believable.  To do so, we will look at the logical structure, focusing on linkage 1 and the conclusion.  Thus, the way to weaken the conclusion will be to break this linkage i.e. find a new piece of information that suggests that the expectation will not be fulfilled, i.e. the companies’ initiative will not be appreciated by the referred people.
Linkage#1
  • Weakener 1 – What if concentrating on the audio message leads to an equal or higher number of distractions while driving?
    • Instance: As the audio system reads texts as and when they are received, a person has to listen to the texts in all situations even if he/ she would have ignored reading a message in some of those scenarios.  
  • Weakener 2 – What if the audio systems lack the much sought after feature of privacy provided by texting and hence leads to an even greater degree of distraction while driving since now the message is shared with all the passengers in the car? In such a case, it’s highly doubtful that the initiative will be appreciated.
With this understanding in mind, let’s take a look at the option statements.
Answer Choices
A
The price of automobiles fitted with such audio-systems is expected to be higher than the price of automobiles not fitted with such systems.
Incorrect -  Irrelevant
The aim is to prevent accidents due to texting. Price of automobiles is not a consideration in the argument.
B
Consumers are generally apprehensive of any major alteration in the standard features offered in an automobile.
Incorrect - Irrelevant
As we don’t know whether installing audio systems would require a major alteration, this choice is not relevant.
C
Studies have shown that five seconds is the average time a driver’s eyes are off the road while texting.
Incorrect - Irrelevant
This option is not relevant to the companies’ initiative, which deals with an alternative to the kind of texting mentioned in this choice.
D
In the past five years, most innovations in the auto-industry have received only a lukewarm response from industry experts.
Incorrect - Irrelevant
The option doesn’t tell us the response of industry experts on this particular innovation. More importantly, the expectation of the auto companies deals with people concerned with the issue and not just industry experts.
E
It is a commonly known fact that an individual driving a vehicle can lose focus by either a lapse in visual attention or an audio distraction.
Correct
This option is written along the lines of our pre-thinking linkage weakener 1.
 

A recent study shows that the overall sales of print per capita has declined at an average 5% per capita over the past decade proving that overall interest in reading has steadily declined.
Internet Geek: People get most of the information online and do not need to spend money on print.
Literary Scholar: Shakespeare still remains the most sold author and his books sell very well. However, today's authors produce few works of similar quality. We need more authors like Shakespeare to re-ignite print sales.
Describe the position taken by Internet Geek and Literary Scholar
  • a)
    The Internet Geek supports the conclusion and provides an explanation for the observed phenomena while the Literary scholar refutes the conclusion and provides an alternate hypothesis.
  • b)
    The Internet Geek provides and alternate explanation to an observed phenomena while the Literary Scholar provides a recommendation and supports the conclusion
  • c)
    The Internet Geek and The Literary Scholar support the conclusion providing a supporting evidence  from their respective fields
  • d)
    The Internet Geek provides the circumstances under which the conclusion could be true while the Literary scholar contrasts the writing styles of 2 eras.
  • e)
    The Internet Geek challenges the premise of the conclusion while the Literary scholar suggests solutions to address the stated fact.
Correct answer is option 'B'. Can you explain this answer?

Rhea Gupta answered
Argument Analysis
 
Pre-Thinking
  1. Internet Geek refutes the conclusion by providing alternate reason for the decline in print sales.
  2. Literary Scholar supports the conclusion and provides reason for the decline in reading.  He also suggest solution to re-ignite the sales.
Answer Choices
A
The Internet Geek supports the conclusion and provides an explanation for the observed phenomena while the Literary scholar refutes the conclusion and provides an alternate hypothesis.
Only the Literary Scholar supports the conclusion.  The Internet Geek does not support the conclusion.  He presents alternate explanation for the observed phenomena.    All portions marked in red are incorrect.
B
The Internet Geek provides and alternate explanation to an observed phenomena while the Literary Scholar provides a recommendation and supports the conclusion
Correct choice. 
C
The Internet Geek and The Literary Scholar support the conclusion providing a supporting evidence  from their respective fields
Only the Literary Scholar supports the conclusion.  Also none of them provide supporting evidence.  The Literary Scholar provides reasoning and solution for the observed phenomenon.
D
The Internet Geek provides the circumstances under which the conclusion could be true while the Literary scholar contrasts the writing styles of 2 eras.
Internet Geek does not support the conclusion in any circumstance.   We may consider the comparison stated by Literary Scholar as contrasting the writing styles of Shakespearean era and modern era.
E
The Internet Geek challenges the premise of the conclusion while the Literary scholar suggests solutions to address the stated fact.
Internet Geek does not challenge the premise – the decline in sales of print.  He challenges the conclusion.

Following a major mishap, a pilgrim spot in the country had been kept closed for the past five years. Recently, a directive by the government has allowed the spot to be reopened for pilgrimage. Since the directive was first published in national newspapers, a huge rush of people, mostly in their early twenties, has been witnessed across the booking offices that have exclusive rights to offer tours to the spot. Clearly, this observation goes against the claim that such spots are most likely to attract pilgrims over the age of 50.
 Which of the following statements most demonstrates a flaw in the argument?
  • a)
    The author fails to consider that the number of tickets booked this year may differ from the number of pilgrims who travelled to the spot five years back.
  • b)
    The author fails to consider that the regional booking offices can also offer tickets to other pilgrim spots.
  • c)
    The author fails to distinguish between people who book tickets from the counters and the ones who travel.
  • d)
    The author fails to consider that the religious sentiments of the young generation can change in five years.
  • e)
    The author does not supply any information on the proportion of booked tickets that end up getting cancelled in the last moment.
Correct answer is option 'C'. Can you explain this answer?

Argument Analysis
Pre-Thinking
Conclusion Clarification
The conclusion states that this observation goes against the claim that such spots are most likely to attract pilgrims over the age of 50. The observation is that people in their early twenties are the ones making the reservations at the offices. 
Pre-Thinking Approach
Let’s see how we can find a flaw in the argument.  To do so, we will look at the logical structure, focusing on linkage 1 and the conclusion. Thus the way to look for a flaw in the argument is to understand whether the author uses sound logic to arrive at the conclusion.
Linkage#1Since the directive was first published in national newspapers, a huge rush of people, mostly in their early twenties, has been witnessed across the booking offices that have exclusive rights to offer tours to the spot.
But what if the people booking the tickets are different from the ones going on the pilgrimage? What if young people are booking tickets for their parents or grandparents? In other words, the author’s argument assumes that:
The people making the bookings are the ones going on the pilgrimage.
However, such an assumption is unwarranted. As stated earlier, it is possible that the young people observed at the booking offices were making reservations for elderly members of their families.
With this pre-thinking in mind, let’s evaluate the answer choices.
Answer Choices
The author fails to consider that the number of tickets booked this year may differ from the number of pilgrims who travelled to the spot five years back.
Incorrect - Irrelevant
There is no comparison between the two time-periods. Hence, this information is completely immaterial to the argument.
B
The author fails to consider that the regional booking offices can also offer tickets to other pilgrim spots.
Incorrect - Irrelevant
The author’s argument is not dependent on the number of tickets sold to one particular pilgrim spot but instead on the age-demographics of the people buying tickets at various booking offices.
C
The author fails to distinguish between people who book tickets from the counters and the ones who travel.
Correct
This option is written along the lines of our pre-thinking per linkage 1.
D
The author fails to consider that the religious sentiments of the young generation can change in five years.
Incorrect - Irrelevant
There is no comparison between the two time-periods. Hence, this information is completely immaterial to the argument. 
E
The author does not supply any information on the proportion of booked tickets that end up getting cancelled in the last moment.
Incorrect - Irrelevant
The proportion of booked tickets that get cancelled does not affect the conclusion of the argument. 

Studies have shown that people are more likely to fall ill when they are stressed. Since companies lose out on precious business days when their employees are not well, it is advisable that they implement regular leisure trips to keep them in good health by organizing several group-bonding fun activities that colleagues can engage in. This way, the companies can save on the money lost as a result of the business they lose when their employees take sick-leaves.
Which of the following must be considered in order to determine whether the proposed plan will be a success in achieving its purpose?
  • a)
    Whether companies can organize seminars by motivational speakers who can increase the efficiency of employees
  • b)
    Whether employees would find out of office get-togethers with colleagues de-stressing
  • c)
    Whether the management of such companies would approve the plan to hold leisure trips
  • d)
    Whether there is any chance that despite these trips there will be a few employees who will fall ill.
  • e)
    Whether there are any good locations in the area the employees are based in that can be used for such trips.
Correct answer is option 'B'. Can you explain this answer?

Pre-Thinking
Conclusion Clarification
The conclusion states that the co. can save on the money lost as a result of the business lost when their employees take SL if they follow a recommendation. The recommendation is to implement regular leisure trips to keep employees healthy by organizing several GBFs that colleagues can engage in.
Pre-Thinking Approach
In order to evaluate the plan, we’ll look at the linkages and pre-think assumptions. Any question that can help us evaluate a central assumption made by the author will be the correct evaluating question. To pre-think assumptions, let’s look at the linkages and understand the conditions under which they don’t hold.  The assumptions then will be that these conditions don’t exist. 
Linkage#1
  • Falsification Condition: What if the leisure trips involving group activities do not help in stress reduction? We see that the plan is likely to fail in such a case. Hence, unless we know the impact of leisure activities with colleagues on stress reduction in employees, the proposed plan cannot be justified.
    • Assumption 1: The leisure trips involving several group-bonding fun activities that colleagues can engage in help in stress reduction.
Linkage#2
  • Falsification Condition: What if the proposed leisure trips end up costing the company more business days than it would have lost in sick leaves? If so, the plan would not be successful.
  • Assumption 2: The average days spent in the proposed leisure trips would not be greater than the average days lost by the company on account of the sick leaves taken by its employees.
With this pre-thinking in mind, let’s evaluate the answer choices.
Answer Choices
A
Whether companies can organize seminars by motivational speakers who can increase the efficiency of employees
Incorrect - Irrelevant
The argument focuses on only one way (leisure activities with colleagues) to achieve a result. An alternate way (seminars) to achieve the same result does not help in evaluating the proposed plan.
B
Whether employees would find out of office get-togethers with colleagues de-stressing
Correct
This option is written along the lines of the assumption 1 mentioned in the pre-thinking analysis. If the answer to the question is yes, it would strengthen the proposal. If it’s no, the proposal would be weakened. 
C
Whether the management of such companies would approve the plan to hold leisure trips
Incorrect - Irrelevant
We are concerned with the success of the plan in case it is approved & implemented. Therefore, whether the management will actually approve the plan or not is not our focus.
D
Whether there is any chance that despite these trips there will be a few employees who will fall ill.
Incorrect - Irrelevant
The argument says “...people are more likely to fall ill when they are stressed.” Here, we are only concerned about higher chances of illness because of stress. Therefore, the argument does not rule out the possibility of illness because of other causes. Also, even if a few people still fall ill because of stress despite going on the leisure trips, the plan can still be successful if many employees who go to leisure trips do not fall ill because of stress.
E
Whether there are any good locations in the area the employees are based in that can be used for such trips.
Incorrect - Irrelevant
The location of the trip doesn’t help us assess whether the company will be able to save on the money lost due to employees’ illness. Even if there aren’t any good locations in areas where the employees are based, such trips can be organized at some other locations. 

According to local farmers, the recent significant decline in the yield of Einkorn, a variety of wheat cultivated primarily in Ingrostia, is due to the unfavorable weather conditions last year.  However, a government spokesperson recently made a statement claiming that the yield of Einkorn has declined because of the decline in the usage of pesticides by the local farmers.
Which of the following, if true, most seriously calls the spokesperson’s claim into question?
  • a)
    Weather conditions in Ingrostia have been the same this year as they were last year.
  • b)
    Einkorn has been losing its popularity to another variety of wheat called Durum that requires cheaper seeds and fewer fertilizers.
  • c)
    The yield of Spelt, a variety of wheat grown in Ingrostia that requires the same weather conditions as Einkorn but is extremely resistant to pests has also declined.
  • d)
    The spokesperson is not from an agricultural background, whereas the farmers are more knowledgeable about agricultural conditions.
  • e)
    The global yield of Einkorn has been lower this year than in the previous years.
Correct answer is option 'C'. Can you explain this answer?

Niharika Sen answered
Argument Analysis
Pre-Thinking
Conclusion Clarification
There is no given conclusion by the author. However, the question stem asks us to weaken the spokesperson’s claim. Hence, we’ll consider this as the conclusion. In the conclusion, a government spokesperson claims that the YOE has declined because of the decline in the use of pesticides. This conclusion challenges the belief of local farmers that unfavorable weather conditions last year are responsible for the decline in YOE.
Pre-Thinking Approach
Let’s see how we can make the conclusion less believable.  To do so, we will look at the logical structure, focusing on linkage 1 and the conclusion.  We see that the conclusion is drawn regarding the cause of the decline in the YOE. Thus, the way to weaken the conclusion will be to weaken this linkage and causal relationship.
Linkage#1 (ref. logical structure)
  • Weakener1 – Since the conclusion of the spokesperson counters the belief of the farmers, an option that increases our belief in the stated cause by the farmers will weaken the spokesperson’s conclusion.
  • Weakener2 – What if even with a decline in the use of pesticides, it was found that the farmers were using enough pesticides? Surely then, the spokesperson’s conclusion will be weakened.
    • Instance 1: Independent studies have shown that farmers in Ingrostia had been using excessive pesticides for years and the recent decline in this usage was a conscious effort to use only sufficient amount of pesticides.
With this understanding in mind, let’s take a look at the option statements.
Answer Choices
A
Weather conditions in Ingrostia have been the same this year as they were last year.
Incorrect - Irrelevant
Knowing that the same weather conditions have persisted this year too doesn’t tell us whether the weather conditions last year are responsible for the decline in YOE.
B
Einkorn has been losing its popularity to another variety of wheat called Durum that requires cheaper seeds and fewer fertilizers.
Incorrect - Irrelevant
Declining popularity of Einkorn doesn’t give us any information about the cause of decline in YOE.
C
The yield of Spelt, a variety of wheat grown in Ingrostia that requires the same weather conditions as Einkorn but is extremely resistant to pests has also declined.
Correct
This choice is written along the lines of our pre-thinking per Linkage weakener 1.  It supports the view of the local farmers while suggesting that insufficient use of pesticides is not likely to be the cause for the decline in YOE.
D
The spokesperson is not from an agricultural background, whereas the farmers are more knowledgeable about agricultural conditions.
Incorrect - Irrelevant
The background of the person making the claim doesn’t give us any information about the correctness of that claim.
E
The global yield of Einkorn has been lower this year than in the previous years.
Incorrect - Irrelevant
This option only highlights the global impact on the YOE; however, it gives us no information about the cause of such a decline.

TV is the future because it remains king of all media. While handsets get hyped, the typical U.S. consumer watches more than 5 hours of TV a day, according to Nielsen, and even younger adults 18 to 24 years old—the supposed digital generation—spend 3 hours and 30 minutes on televisions daily compared to only 49 minutes on the Web and 20 minutes on mobile. 
The above argument is most vulnerable to which of the following criticisms?
  • a)
    The argument fails to look into the possible causes of higher consumption of TV over other media
  • b)
    The argument makes an assumption that the consumer preferences will not change significantly over time without looking at the past trends.
  • c)
    The argument fails to consider that the average figures may not apply to every consumer of media.
  • d)
    The argument does not talk about population below 18 years of age, which will soon be part of the so called “digital generation”
  • e)
    The argument proposes that the same cause and effect may apply in the future as in the past without even looking at the possibility of existence of cause in the future.
Correct answer is option 'B'. Can you explain this answer?

Arjun Iyer answered
Analysis of Vulnerability:

Assumption of Consumer Preferences:
The argument assumes that consumer preferences will remain consistent over time without considering past trends. This is a vulnerability because consumer preferences can shift rapidly, especially with advancements in technology and changes in societal behaviors.
Therefore, the argument is most vulnerable to the criticism that it makes an assumption about consumer preferences without taking into account the potential for significant changes in the future. The argument does not provide evidence or analysis to support the claim that TV will continue to dominate over other forms of media in the long term.

Which of the following affects capital budgeting decision?
  • a)
    Investment Criteria and interest rate
  • b)
    Rate of Return
  • c)
    Cash Flow of the Project
  • d)
    All of these
Correct answer is option 'D'. Can you explain this answer?

Tanvi Roy answered
Overview:
Capital budgeting refers to the process of evaluating and selecting long-term investment projects that are expected to generate cash flows over a specified period. It involves analyzing the potential returns and risks associated with different investment opportunities and determining whether they are financially viable. There are several factors that affect capital budgeting decisions, and these include investment criteria, interest rate, rate of return, and cash flow of the project.

Investment Criteria:
Investment criteria are the set of standards or benchmarks that companies use to evaluate and select investment projects. These criteria typically include methods such as payback period, net present value (NPV), internal rate of return (IRR), and profitability index. Each of these criteria considers different aspects of the investment, such as the time it takes to recover the initial investment, the profitability of the project, and the value of the future cash flows. The selection of investment criteria will depend on the company's objectives, risk tolerance, and other specific factors.

Interest Rate:
The interest rate is a key factor in capital budgeting decisions as it directly affects the cost of capital. The cost of capital is the minimum rate of return that a company requires to undertake an investment project. It represents the opportunity cost of using the company's funds for a particular investment instead of other investment alternatives or debt repayment. A higher interest rate increases the cost of capital, making it more difficult for projects to meet the required rate of return and thus impacting the decision to invest.

Rate of Return:
The rate of return is another important factor in capital budgeting decisions. It represents the expected profitability of an investment project and is typically measured by metrics such as the IRR or the return on investment (ROI). The rate of return is compared to the company's required rate of return or hurdle rate to determine whether the project is financially feasible. If the expected rate of return is higher than the required rate of return, the project may be considered acceptable.

Cash Flow of the Project:
The cash flow of the project is a crucial factor in capital budgeting decisions. It includes both the initial investment outlay and the expected future cash flows generated by the project. The cash flows are estimated by considering factors such as sales revenue, operating expenses, taxes, and working capital requirements. The timing and magnitude of the cash flows are important as they determine the project's profitability and the potential for creating value for the company. The cash flows are discounted to their present value using the cost of capital to calculate metrics such as the NPV and IRR.

Conclusion:
In conclusion, capital budgeting decisions are influenced by various factors, including investment criteria, interest rate, rate of return, and the cash flow of the project. Companies must carefully evaluate these factors to determine the financial viability of investment opportunities and make informed decisions that align with their strategic objectives and financial goals. By considering these factors, companies can optimize their capital allocation and enhance their long-term profitability.

The Observer is a newspaper company that has two divisions: The East Observer and The West Observer. All the employees at The Observer work in either of the two divisions. For the past five years, The East Observer has consistently accounted for 40% of the total workforce of The Observer while The West Observer has accounted for the rest. Over the same period, The West Observer has accounted for 50% of the total wages paid by The Observer.
Which of the following can properly be inferred about the past five years from the statements above?
  • a)
    The distribution of people across different hierarchical levels is different in The East Observer than in The West Observer.
  • b)
    The average motivation level of employees working in the two divisions is different, even though the difference in number is not significantly large.
  • c)
    The average revenue generated per employee is expected to be greater in The East Observer than in The West Observer.
  • d)
    The average wage paid to the workforce is greater at The West Observer than at The East Observer.
  • e)
    The number of people working at The East Observer is at least 25% less than the number of people working at The West Observer.
Correct answer is option 'E'. Can you explain this answer?

Parth Singh answered
Pre-Thinking
Conclusion Clarification
N/ A
Pre-Thinking Approach
In order to see what can be inferred about the past five years from the statements, we need to observe all the facts given in the argument and see if the information in the passage is true, what else must also be true.
Inferences: – From the above information we can infer the following:
  • Fact #3 – For the past five years, The EO has consistently accounted for 40% of the total workforce of The Observer while The West Observer has accounted for the rest.
  • Inference – For the past five years, The WO accounted for 60% of the workforce of The Observer.
 
  • Fact #4 – Over the same period, The WO has accounted for 50% of the total wages paid by The Observer.
  • Inference –Fore the past five years, the EO has accounted for 50% of the total wages paid by The Observer.
 
  • Linkage #3 – (ref. logical structure)
  • Inference –The average pay at EO was 50% higher (1.25/0.83 = 1.5) than the average pay at WO.
For example: Let’s assume that the total wages paid were $100 while the total number of employees at The Observer was 100.  This means, the average pay at WO was $50/60 = $0.83, while the average pay at EO was $50/40 = $1.25.
With this pre-thinking in mind, let's take a look at the option statements.
Answer Choices
A
The distribution of people across different hierarchical levels is different in The East Observer than in The West Observer.
Incorrect - No information given
This option can’t be inferred from the passage since we have no information regarding hierarchy.
B
The average motivation level of employees working in the two divisions is different, even though the difference in number is not significantly large.
Incorrect - No information given
This option can’t be inferred from the information given. The passage doesn’t talk about motivation level at all.
C
The average revenue generated per employee is expected to be greater in The East Observer than in The West Observer.
Incorrect - No information given
The passage doesn’t given any information about revenue generation.
D
The average wage paid to the workforce is greater at The West Observer than at The East Observer.
Incorrect - Against the given information
This option is opposite of statement 5, which we arrived at, during pre-thinking.
E
The number of people working at The East Observer is at least 25% less than the number of people working at The West Observer.
Correct
This option is written along the lines of our pre-thinking linkage 3. If there were 60 employees at WO, there would be 40 at EO. This means that the employee strength at EO is 40/60 ~= 66% of that at WO. Or in other words, there are nearly 33% less employees at The East Observer, compared to The West Observer. This number is higher than 25% as mentioned in this option.
 

A film is more likely to have a complex storyline when it is directed by a director who has won an Academy Award. Regardless of the awards won by the director, a film with a complex storyline is more likely to do badly at the box-office than a film with a story-line that is not complex. Since other causes of poor box-office performance are not related to the awards won by the director, there must be more films with poor box-office collections with Academy Award-winning directors than with directors who have not won any Academy Awards.The argument above is most vulnerable to which one of the following criticisms?
  • a)
    It ignores the fact that a film that does not do well on the box-office may eventually do well when broadcasted on TV.
  • b)
    It fails to consider a scenario in which the number of films directed by directors who have not won Academy Awards is significantly more than that directed by directors who have won Academy Awards.
  • c)
    It identifies one aspect of the films as the sole determinant of their box office performance without considering other possible aspects.
  • d)
    It fails to consider a scenario in which directors may be aware of the fact that a complex storyline may not translate into a successful box-office collection.
  • e)
    It fails to factor in the number of films with poor box-office collections that were directed by Academy Award-winning directors and that ultimately won critical acclaim.
Correct answer is option 'B'. Can you explain this answer?

Mihir Nambiar answered
Argument Analysis
 
Pre-Thinking
Conclusion Clarification
The author concludes that total number of films with poor BO collections directed by AWDs would be greater than total number of films with poor BO collections directed by NAWDs. The conclusion is based on the fact that AWDs are more likely to direct a film with CS, and irrespective of the awards won by the director, films with CS are more likely to perform worse at box office than films without CS. All other factors behind poor performance of films at box office are unrelated to awards won by the directors.
 Pre-Thinking Approach
To pre-think a flaw in the reasoning, let’s see how we can falsify this conclusion.  To do so, we will look at the logical structure, focusing on linkage 1 and the conclusion.  We will understand the condition under which the conclusion is falsified. Thus, the flaw would show a logical gap in the author’s reasoning while reaching the conclusion.
Linkage#1
  • Falsification Condition:  What if the total number of films directed by NAWDs is significantly more than that directed by AWDs? In such a case, there may not be more films with poor BO collections with AWDs than with NAWDs.
  • Flaw:  The argument is vulnerable to the criticisms that there is an abrupt jump from the premises given to the conclusion drawn. The argument has arrived at the conclusion without comparing the actual total number of films made by Academy Award winning directors and the ones made by the rest of the directors.
To understand the same, please consider the following scenario:
 
With this pre-thinking in mind, let’s evaluate the answer choices.
Answer Choices
A
It ignores the fact that a film that does not do well on the box-office may eventually do well when broadcasted on TV.
Incorrect - No flaw
This statement is of no relevance to the argument since the argument is concerned with the performance of the films only at the box office.
B
It fails to consider a scenario in which the number of films directed by directors who have not won Academy Awards is significantly more than that directed by directors who have won Academy Awards.
Correct
This choice is written along the lines of our pre-thinking per Linkage #1. It points out that the argument does not compare the actual total number of films made by AWDs and NAWDs. If we see the pre-thinking example, we can notice that the author’s conclusion is incorrect for the particular scenario.
C
It identifies one aspect of the films as the sole determinant of their box office performance without considering other possible aspects.
Incorrect - No flaw
This statement is incorrect since the passage does talk about “other factors” besides a complex storyline that determine a film’s performance at the box office. The passage says that all other factors are not related to awards won by directors.
D
It fails to consider a scenario in which directors may be aware of the fact that a complex storyline may not translate into a successful box-office collection.
Incorrect - No flaw
The awareness of directors regarding the eventual box-office performance of their films is of no relevance to the author’s conclusion.
E
It fails to factor in the number of films with poor box-office collections that were directed by Academy Award-winning directors and that ultimately won critical acclaim.
Incorrect - No flaw
This statement is irrelevant since the author’s conclusion is concerned with only the box-office performance of such films.

Composite materials are synthetic materials made by combining multiple materials that have differing chemical compositions, and the resultant material has chemical properties that differ significantly from those of any of the original materials, and is stronger, less heavy, and less expensive than conventional materials. While composites have been commonly used throughout history to construct building materials, they are actually predated by natural composites such as shellac, which are cheaper than synthetic materials. Therefore, to minimize expenses, natural composites should be used whenever possible.
Which of the following would it be most useful to know in determining whether using natural composites would minimize expenses?
  • a)
    Whether some natural composites cost more than synthetic composites
  • b)
    Whether the long term costs of maintaining buildings made using natural composites is significantly higher than those made using synthetic composites
  • c)
    Whether some composites have properties that do not differ significantly from the properties of the base materials used to create the composite
  • d)
    Whether either natural or synthetic composites have uses in markets other than the one for building materials
  • e)
    Whether there are any natural composites that predate shellac
Correct answer is option 'B'. Can you explain this answer?

Arya Yadav answered
Pre-Thinking
Conclusion Clarification
The plan of using NC would achieve the goal of minimizing expenses.
Pre-Thinking Approach
Let’s ask questions which would help us in examining linkage 1. We’ll think of scenarios where the conclusion isn’t valid and ask whether such scenarios exist or not.
Scenario 1: What if, even though NC are cheaper, it costs more to maintain buildings made using NC? The overall cost in this situation can be much higher. If that’s the case, we can’t conclude that the use of NC would minimize expenses.
With this understanding in mind, let’s take a look at the option statements.
Answer Choices
A
Whether some natural composites cost more than synthetic composites
Incorrect - Irrelevant
The cost of some NCs isn’t relevant as we know that there are other NCs which are cheaper than CM.
B
Whether the long term costs of maintaining buildings made using natural composites is significantly higher than those made using synthetic composites
Correct
This choice is written along the lines of our pre-thinking per Scenario 1.
C
Whether some composites have properties that do not differ significantly from the properties of the base materials used to create the composite
Incorrect - Irrelevant
A comparison of the properties of composites with those of the base materials doesn’t tell us whether the use of NC will be cheaper or not.
D
Whether either natural or synthetic composites have uses in markets other than the one for building materials
Incorrect - Irrelevant
We aren’t concerned with other uses of composites.
E
Whether there are any natural composites that predate shellac
Incorrect - Irrelevant
The existence of such materials doesn’t tell us whether expenses would be minimized by using NC.

A new breed of computer viruses called Super-V are quite sophisticated in the sense that they can cause irreversible damage to data files on the computer. Hard drive of computer infected with such a virus is formatted to get rid of the virus. Since this virus cannot magically re-appear in the formatted areas, any subsequent damage caused to the data files after removal of such virus is not because of this Super-V virus.
Which of the following, if true, most seriously weakens the conclusion above?
  • a)
    The main indications of Super-V virus are changes in sizes and contents of the data-files in the infected computer.
  • b)
    Some online virus-removal tools provided by reputed anti-virus program developers does do a good job of deleting this virus from the computer.
  • c)
    A few other indications of Super-V virus other than changes in size and contents of data-files in the computer can surface if anti-virus program is disabled on the computer.
  • d)
    Sometimes the virus hides itself in the boot sector of the hard drive that is not touched during the formatting process.
  • e)
    It is quite possible that the size and content of some data files on the infected computer may not change.
Correct answer is option 'D'. Can you explain this answer?

Niharika Sen answered
Pre-Thinking
1. Assumptions:
  • Formatting the hard drive completely removes the virus.
  • Virus cannot copy itself from the unformatted areas to formatted areas.
2. Weakeners
  • Evidence suggesting virus is not completely removed when the hard drive is formatted.
  • Evidence suggesting that virus can be copied from unformatted areas to formatted areas.
Answer Choices
A
The main indications of Super-V virus are changes in sizes and contents of the data-files in the infected computer.
This choice merely states what the virus does and does not provide evidence that states that Super-V could damage files after formatting. Therefore, this choice is OFS.
B
Some online virus-removal tools provided by reputed anti-virus program developers does do a good job of deleting this virus from the computer.
This choice describes that some tools remove the virus completely and in that regard support the conclusion that any subsequent damage should not be attributed to previous infection by Super-V.
C
A few other indications of Super-V virus other than changes in size and contents of data-files in the computer can surface if anti-virus program is disabled on the computer.
This choice describes how the virus surfaces in the absence of an anti-virus program and does not touch the central issue. Therefore, this choice is OFS.
D
Sometimes the virus hides itself in the boot sector of the hard drive that is not touched during the formatting process.
This choice provides evidence that the virus is not completely removed during the formatting process and remains on the hard-drive. Therefore, after formatting, there is a probability that this virus may infect the hard drive by moving from the non-formatted area to the formatted area. Therefore, this is the correct answer. 
E
It is quite possible that the size and content of some data files on the infected computer may not change.
This choice describes how the virus surfaces in the infected computer (in absence of an anti-virus program) and does not touch the central issue. Therefore, this choice is OFS.
 

Chapter doubts & questions for Introduction - 35 Days Preparation for GMAT 2025 is part of GMAT exam preparation. The chapters have been prepared according to the GMAT exam syllabus. The Chapter doubts & questions, notes, tests & MCQs are made for GMAT 2025 Exam. Find important definitions, questions, notes, meanings, examples, exercises, MCQs and online tests here.

Chapter doubts & questions of Introduction - 35 Days Preparation for GMAT in English & Hindi are available as part of GMAT exam. Download more important topics, notes, lectures and mock test series for GMAT Exam by signing up for free.

35 Days Preparation for GMAT

171 videos|269 docs|181 tests

Top Courses GMAT